2018 questions
Quiz-summary
0 of 44 questions completed
Questions:
- 1
- 2
- 3
- 4
- 5
- 6
- 7
- 8
- 9
- 10
- 11
- 12
- 13
- 14
- 15
- 16
- 17
- 18
- 19
- 20
- 21
- 22
- 23
- 24
- 25
- 26
- 27
- 28
- 29
- 30
- 31
- 32
- 33
- 34
- 35
- 36
- 37
- 38
- 39
- 40
- 41
- 42
- 43
- 44
Information
2018 Board Questions
You have already completed the quiz before. Hence you can not start it again.
Quiz is loading...
You must sign in or sign up to start the quiz.
You have to finish following quiz, to start this quiz:
Results
0 of 44 questions answered correctly
Your time:
Time has elapsed
You have reached 0 of 0 points, (0)
Categories
- Not categorized 0%
-
2018 questions
- 1
- 2
- 3
- 4
- 5
- 6
- 7
- 8
- 9
- 10
- 11
- 12
- 13
- 14
- 15
- 16
- 17
- 18
- 19
- 20
- 21
- 22
- 23
- 24
- 25
- 26
- 27
- 28
- 29
- 30
- 31
- 32
- 33
- 34
- 35
- 36
- 37
- 38
- 39
- 40
- 41
- 42
- 43
- 44
- Answered
- Review
-
Question 1 of 44
1. Question
A 24 year-old male presents to the office with left index finger pain for one week. The pain seemed like it came out of nowhere when he was moving some books. He denies prior injury to the finger. The tip of his finger has continued to bother him to the touch and with any motion. On exam he has tenderness to the distal phalanx without deformity. AP and lateral x-rays of the finger (figures 1 and 2) show a nondisplaced fracture at the base of the distal phalanx through a radiolucent lesion. What is the next best step in treatment for this patient?
Correct
The patient presents with a well- defined centrally located radiolucent lesion with corticol thinning and stippled calcification. The lesion was asymptomatic prior to the patient’s pathological fracture. This presentation is consistent with an enchondroma, the most common tumor of the hand. X-rays of the lesion and patient history make the diagnosis of enchondroma in most cases. If the diagnosis is unclear, an MRI may be ordered. Indications for MRI include increasing pain, large aggressive appearing lesions, corticol destruction, and surrounding soft tissue extension. The vast majority of enchondromas are found incidentally on x-ray and can be treated with observation. Routine follow-up x-rays are generally not recommended unless the patient becomes symptomatic, the lesion appears aggressive, or there is a risk for pathological fracture. Patients who do present with a pathological fracture should be treated with immobilization for 6 weeks until the fracture heals. Surgery is recommended once the fracture heals as the lesion is prone to future fracture. Surgery involves intralesional curettage and bone grafting which fills the void and helps reduce the risk of future pathological fracture. Bone biopsy is taken at the time of curettage to confirm the diagnosis. 1,2
References
1. Enchondroma. http://www.orthobullets.com. Accessed on 5/17/17.
2. Lubahn JD, Bachoura A. Enchondroma of the Hand: Evaluation and Management. JAAOS 2016; 24: 625-633.Incorrect
The patient presents with a well- defined centrally located radiolucent lesion with corticol thinning and stippled calcification. The lesion was asymptomatic prior to the patient’s pathological fracture. This presentation is consistent with an enchondroma, the most common tumor of the hand. X-rays of the lesion and patient history make the diagnosis of enchondroma in most cases. If the diagnosis is unclear, an MRI may be ordered. Indications for MRI include increasing pain, large aggressive appearing lesions, corticol destruction, and surrounding soft tissue extension. The vast majority of enchondromas are found incidentally on x-ray and can be treated with observation. Routine follow-up x-rays are generally not recommended unless the patient becomes symptomatic, the lesion appears aggressive, or there is a risk for pathological fracture. Patients who do present with a pathological fracture should be treated with immobilization for 6 weeks until the fracture heals. Surgery is recommended once the fracture heals as the lesion is prone to future fracture. Surgery involves intralesional curettage and bone grafting which fills the void and helps reduce the risk of future pathological fracture. Bone biopsy is taken at the time of curettage to confirm the diagnosis. 1,2
References
1. Enchondroma. http://www.orthobullets.com. Accessed on 5/17/17.
2. Lubahn JD, Bachoura A. Enchondroma of the Hand: Evaluation and Management. JAAOS 2016; 24: 625-633. -
Question 2 of 44
2. Question
A 34 year-old male presents to the emergency department after a motor vehicle accident with severe left hip pain and a lower extremity rotational deformity. He was the driver when he lost control and hit a pole. On exam he has a shortened and externally rotated left lower extremity. X-rays of the left hip show a posterior hip dislocation. What is the best treatment option for this patient?Correct
Traumatic hip dislocations are caused by a high energy injury such as a motor vehicle accident. A posterior hip dislocation can occur as the flexed knee hits the dashboard and the femur is pushed out of the posterior pelvis. Posterior hip dislocations account for 90% of all traumatic hip dislocations. A posterior hip dislocation typically occurs in younger patients as the soft tissues give way before bone is fractured. Older patients will typically sustain a fracture before a dislocation occurs. A simple dislocation is a dislocation without fracture where a complex dislocation occurs with a fracture. 1
Standard pre and post reduction x-rays are required for posterior dislocations. CT should be ordered post reduction to determine if any loose bodies or associated fractures occurred. A careful neurovascular exam should be documented after reduction to rule out sciatic nerve injury which occurs in 10-20% of hip dislocations. Ipsilateral knee injuries can occur in up to 25% of patients. MRI arthrogram may be ordered in an outpatient setting if the patient has continued pain beyond 4-6 weeks. MRI helps identify associated labral tears, chondral lesions, and osteonecrosis. In American Professional Football players who sustain a posterior dislocation an MRI is done at 6 weeks to rule out osteonecrosis before return to play clearance. A period of 4-6 weeks of protected weight bearing followed by a gradual return to activities is the typical treatment protocol for simple hip dislocations. 1,2
Answer A
References
1. Hip Dislocation. http://www.orthobullets.com. Accessed on 2/24/2018.
2. Moorman CT 3rd1, Warren RF, Hershman EB, Crowe JF, Potter HG, Barnes R, O’Brien SJ, Guettler JH. Traumatic posterior hip subluxation in American football. J Bone Joint Surg Am. 2003 Jul;85-A(7):1190-6.Incorrect
Traumatic hip dislocations are caused by a high energy injury such as a motor vehicle accident. A posterior hip dislocation can occur as the flexed knee hits the dashboard and the femur is pushed out of the posterior pelvis. Posterior hip dislocations account for 90% of all traumatic hip dislocations. A posterior hip dislocation typically occurs in younger patients as the soft tissues give way before bone is fractured. Older patients will typically sustain a fracture before a dislocation occurs. A simple dislocation is a dislocation without fracture where a complex dislocation occurs with a fracture. 1
Standard pre and post reduction x-rays are required for posterior dislocations. CT should be ordered post reduction to determine if any loose bodies or associated fractures occurred. A careful neurovascular exam should be documented after reduction to rule out sciatic nerve injury which occurs in 10-20% of hip dislocations. Ipsilateral knee injuries can occur in up to 25% of patients. MRI arthrogram may be ordered in an outpatient setting if the patient has continued pain beyond 4-6 weeks. MRI helps identify associated labral tears, chondral lesions, and osteonecrosis. In American Professional Football players who sustain a posterior dislocation an MRI is done at 6 weeks to rule out osteonecrosis before return to play clearance. A period of 4-6 weeks of protected weight bearing followed by a gradual return to activities is the typical treatment protocol for simple hip dislocations. 1,2
Answer A
References
1. Hip Dislocation. http://www.orthobullets.com. Accessed on 2/24/2018.
2. Moorman CT 3rd1, Warren RF, Hershman EB, Crowe JF, Potter HG, Barnes R, O’Brien SJ, Guettler JH. Traumatic posterior hip subluxation in American football. J Bone Joint Surg Am. 2003 Jul;85-A(7):1190-6. -
Question 3 of 44
3. Question
A 45 year-old female presents to the office with a history of left heel pain for several months. She has a past medical history of diabetes, high cholesterol, hypertension, and depression. Her pain is made worse with running and stairs. Recent x-rays taken include an AP mortise and lateral view of her ankle show no abnormalities. She has been diagnosed with Achilles tendonitis and has tried ice, NSAIDs, rest, and physical therapy without much relief. Which medication could be contributing to her tendonitis?
Correct
Statins, along with fluoroquinolones, are known to cause systemic side effects including tendinitis and tendon rupture. For patients with hypercholesterolemia, a statin is by far the most common medication used. Statins are the most effective medication in reducing LDL cholesterol which reduces risk of coronary artery disease, stroke, and death. Statins can cause a number of side effects including myalgias, myopathy, tendinitis, and elevated liver enzymes. The etiology of why statins cause tendinopathy is unclear, although the most widely accepted theory is that statins interfere with healing after microtrauma to the tendon. The Achilles tendon is the most susceptible to injury due to size and repetitive use. In effected patients, most will experience symptoms within four months after starting a statin. Symptoms usually resolve within 2-3 weeks after stopping the statin. Bottom line: patients at higher risk of tendinitis, including those with diabetes, hyperuricemia, and who participate in sports, should be monitored closely while on a statin. 1,2
Answer D.
References
1. Deren ME, Klinge SA, Mukand NH, Mukand JA. Tendinopathy and Tendon Rupture Associated with Statins. JBJS Reviews 2016; 4 (5). 1-6.
2. 1.Marie I, Delafenêtre H, Massy N, et al. Tendinous disorders attributed to statins: A study on ninety-six spontaneous reports in the period 1990-2005 and review of the literature. Arthritis Care Res. 2008;59:367-372.Incorrect
Statins, along with fluoroquinolones, are known to cause systemic side effects including tendinitis and tendon rupture. For patients with hypercholesterolemia, a statin is by far the most common medication used. Statins are the most effective medication in reducing LDL cholesterol which reduces risk of coronary artery disease, stroke, and death. Statins can cause a number of side effects including myalgias, myopathy, tendinitis, and elevated liver enzymes. The etiology of why statins cause tendinopathy is unclear, although the most widely accepted theory is that statins interfere with healing after microtrauma to the tendon. The Achilles tendon is the most susceptible to injury due to size and repetitive use. In effected patients, most will experience symptoms within four months after starting a statin. Symptoms usually resolve within 2-3 weeks after stopping the statin. Bottom line: patients at higher risk of tendinitis, including those with diabetes, hyperuricemia, and who participate in sports, should be monitored closely while on a statin. 1,2
Answer D.
References
1. Deren ME, Klinge SA, Mukand NH, Mukand JA. Tendinopathy and Tendon Rupture Associated with Statins. JBJS Reviews 2016; 4 (5). 1-6.
2. 1.Marie I, Delafenêtre H, Massy N, et al. Tendinous disorders attributed to statins: A study on ninety-six spontaneous reports in the period 1990-2005 and review of the literature. Arthritis Care Res. 2008;59:367-372. -
Question 4 of 44
4. Question
A 17 year-old male presents to the office with right elbow pain for several months. He plays baseball 7 months out of the year and either pitches or plays third base. His pain is made worse when throwing and he has lost velocity on his fastball. Ice and NSIADs provide little relief. He tried taking a few months off from pitching but the pain returned as he started throwing again. On exam you note tenderness over the radiocapitellar joint and pain during the lateral compression test. He has no medial sided laxity or pain with valgus stress. AP and lateral x-rays of the right elbow are normal. What is the most likely diagnosis?
Correct
The overhead throwing motion places significant stress on the elbow which can often lead to overuse injury. Determining pathology in the overhead athlete starts with the basic understanding that a valgus force is placed on the elbow during the throwing motion. This causes tensile forces to the medial elbow and compression forces to the lateral elbow. Common sources of medial elbow pain in the throwing athlete are often attributed to a stretch of the medial structures including ulnar collateral ligament tearing, ulnar neuritis, flexor pronator strain, and medial apophysitis. Repetitive compressive forces at the lateral elbow may cause a condition called osteochondritis dissecans (OCD) of the capitellum. 1
OCD of the capitellum occurs when repetitive trauma starts to damage the articular cartilage and subchondral bone of the capitellum. The condition generally occurs between the ages of 11 and 21 and is more common in males. Pain is often described as worse with activities and better with rest, with the occasional catching or locking occurring during throwing motion. On exam tenderness along the radiocapitellar articulation and pain with the lateral compression test is often noted. Radiographs are generally negative early in the disease but may show irregularity to the articular surface as the disease advances. MRI is the most sensitive study for picking up OCD early. Early stable OCD can be treated successfully with 3-6 weeks of rest followed by a gradual return to throwing. Large or unstable OCD lesions often require operative debridement and lesion excision, debridement and bone marrow stimulation, or fragment fixation. (Note: Panner disease occurs almost exclusively in males under 10 years of age and is not associated with trauma.) 1,2
References
1. Patel RM, Lynch TS, Amin NH, Gryzlo S, Schickenantz M. Elbow Injuries in the Throwing Athlete. JBJS Reviews 2014; 2 (11). Pages 1-11.
2. Ruchelsman DE, Hall MP, Youm T. Osteochondritis Dissecans of the Capitellum: Current Concepts. JAAOS 2010; 18: 557-567.Incorrect
The overhead throwing motion places significant stress on the elbow which can often lead to overuse injury. Determining pathology in the overhead athlete starts with the basic understanding that a valgus force is placed on the elbow during the throwing motion. This causes tensile forces to the medial elbow and compression forces to the lateral elbow. Common sources of medial elbow pain in the throwing athlete are often attributed to a stretch of the medial structures including ulnar collateral ligament tearing, ulnar neuritis, flexor pronator strain, and medial apophysitis. Repetitive compressive forces at the lateral elbow may cause a condition called osteochondritis dissecans (OCD) of the capitellum. 1
OCD of the capitellum occurs when repetitive trauma starts to damage the articular cartilage and subchondral bone of the capitellum. The condition generally occurs between the ages of 11 and 21 and is more common in males. Pain is often described as worse with activities and better with rest, with the occasional catching or locking occurring during throwing motion. On exam tenderness along the radiocapitellar articulation and pain with the lateral compression test is often noted. Radiographs are generally negative early in the disease but may show irregularity to the articular surface as the disease advances. MRI is the most sensitive study for picking up OCD early. Early stable OCD can be treated successfully with 3-6 weeks of rest followed by a gradual return to throwing. Large or unstable OCD lesions often require operative debridement and lesion excision, debridement and bone marrow stimulation, or fragment fixation. (Note: Panner disease occurs almost exclusively in males under 10 years of age and is not associated with trauma.) 1,2
References
1. Patel RM, Lynch TS, Amin NH, Gryzlo S, Schickenantz M. Elbow Injuries in the Throwing Athlete. JBJS Reviews 2014; 2 (11). Pages 1-11.
2. Ruchelsman DE, Hall MP, Youm T. Osteochondritis Dissecans of the Capitellum: Current Concepts. JAAOS 2010; 18: 557-567. -
Question 5 of 44
5. Question
A 35 year-old male presents to the office with right hip pain after a twist and split type fall while working construction 3 months ago. He has experienced sharp pain when rotating his hip since the injury. On exam he has pain with hip flexion and internal rotation. AP and lateral x-rays of the hip show no signs of femoroacetabular impingement or arthritis. MRI arthrogram shows a degenerative labral tear. Which statement below is accurate regarding treatment with an arthroscopic labral debridement?
Correct
Hip arthroscopy is an increasingly utilized procedure in orthopedic practice. Debridement of labral tears remains the most common indication for hip arthroscopy. Indications for hip arthroscopy include hip pain beyond one month, positive findings on physical exam, and MRI confirmation of labral pathology. Hip arthroscopy with debridement is an effective treatment option for patients with degenerative labral tears. Arthroscopic debridement with partial labrectomy has been found to reduce hip pain and resolve mechanical symptoms in 91% of patients with labral tears. Poor prognostic indicators after hip arthroscopy include pre-existing arthritis, femoroacetabular impingement, and age over 45 years old. During an arthroscopic labral debridement loose flaps of labrum and calcifications are shaved away leaving as much normal labrum intact as possible. Patients with an unstable degenerative flap type labral tear, like this patient, will likely do well with an arthroscopic debridement. A labral repair is performed only if the labrum is non-viable after debridement.
References
1. Wilkin G, March G, Beaule PE. Athroscopic Acetabular Labral Debridement in Patients Forty Five Years of Age or Older Has Minimal Benefit for Pain and Function. JBJS 2014; 96: 113-8.
2. Domb BG, Hartigan DE, Perets I. Decision Making for Labral Treatment in the Hip: Repair Versus Debridement Versus Reconstruction. JAAOS 2017; 25: e53-e62Incorrect
Hip arthroscopy is an increasingly utilized procedure in orthopedic practice. Debridement of labral tears remains the most common indication for hip arthroscopy. Indications for hip arthroscopy include hip pain beyond one month, positive findings on physical exam, and MRI confirmation of labral pathology. Hip arthroscopy with debridement is an effective treatment option for patients with degenerative labral tears. Arthroscopic debridement with partial labrectomy has been found to reduce hip pain and resolve mechanical symptoms in 91% of patients with labral tears. Poor prognostic indicators after hip arthroscopy include pre-existing arthritis, femoroacetabular impingement, and age over 45 years old. During an arthroscopic labral debridement loose flaps of labrum and calcifications are shaved away leaving as much normal labrum intact as possible. Patients with an unstable degenerative flap type labral tear, like this patient, will likely do well with an arthroscopic debridement. A labral repair is performed only if the labrum is non-viable after debridement.
References
1. Wilkin G, March G, Beaule PE. Athroscopic Acetabular Labral Debridement in Patients Forty Five Years of Age or Older Has Minimal Benefit for Pain and Function. JBJS 2014; 96: 113-8.
2. Domb BG, Hartigan DE, Perets I. Decision Making for Labral Treatment in the Hip: Repair Versus Debridement Versus Reconstruction. JAAOS 2017; 25: e53-e62 -
Question 6 of 44
6. Question
A 64 year-old male presents to the office with right foot pain for several months. He denies having a prior injury or known precipitating event. The pain is worse in the morning and when going down stairs. On exam he has pain to palpation over the distal 2nd metatarsal. AP x-ray of the foot shows mild flattening of the second metatarsal head. MRI shows collapse of the 2nd metatarsal head with subarticular sclerosis and edema. What is the most likely diagnosis?
Correct
Freiberg’s disease is a poorly understood condition in which the articular surface of the second metatarsal head begins to collapse causing pain at the second metatarsophalangeal (MTP) joint. Although less common, the condition can occur in the third and fourth metatarsal heads as well. The cause is thought to be from a disruption in the blood supply of the metatarsal head. This is why the condition is also referred to as a Freiberg’s infarction. On x-ray Freiberg’s starts with a subchondral fracture and collapse (which may only be seen on MRI) and later may progress to further collapse, joint space narrowing, and arthritis. Patients may present initially with little to no changes on x-ray and an MRI is often necessary to establish the diagnosis. Symptoms include forefoot pain that is made worse with activities and localized pain to the second metatarsal head.
Initial treatment aims at taking pressure off the effected metatarsal head. Patients can be placed in a walking boot or use a stiff shoe insert with a metatarsal pad. Activity modification and NSAIDs can help manage symptoms. A metatarsophalangeal (MTP) joint steroid injection may provide temporary relief and is particularly useful with advanced arthritic changes. If conservative treatments fail the most common operative treatment is an MTP joint arthrotomy with removal of loose bodies and osteophytes. The metatarsal head can be drilled or bone grafted to treat the collapsed metatarsal head. Metatarsal head resection is usually avoided as it increases the weight bearing load on the adjacent metatarsal heads.
References
1. Freiberg’s disease. http://www.orthobullets.com. Accessed on 10/21/2017.
2. Freiberg’s disease. http://www.aofas.org. Accessed on 10/21/2017.Incorrect
Freiberg’s disease is a poorly understood condition in which the articular surface of the second metatarsal head begins to collapse causing pain at the second metatarsophalangeal (MTP) joint. Although less common, the condition can occur in the third and fourth metatarsal heads as well. The cause is thought to be from a disruption in the blood supply of the metatarsal head. This is why the condition is also referred to as a Freiberg’s infarction. On x-ray Freiberg’s starts with a subchondral fracture and collapse (which may only be seen on MRI) and later may progress to further collapse, joint space narrowing, and arthritis. Patients may present initially with little to no changes on x-ray and an MRI is often necessary to establish the diagnosis. Symptoms include forefoot pain that is made worse with activities and localized pain to the second metatarsal head.
Initial treatment aims at taking pressure off the effected metatarsal head. Patients can be placed in a walking boot or use a stiff shoe insert with a metatarsal pad. Activity modification and NSAIDs can help manage symptoms. A metatarsophalangeal (MTP) joint steroid injection may provide temporary relief and is particularly useful with advanced arthritic changes. If conservative treatments fail the most common operative treatment is an MTP joint arthrotomy with removal of loose bodies and osteophytes. The metatarsal head can be drilled or bone grafted to treat the collapsed metatarsal head. Metatarsal head resection is usually avoided as it increases the weight bearing load on the adjacent metatarsal heads.
References
1. Freiberg’s disease. http://www.orthobullets.com. Accessed on 10/21/2017.
2. Freiberg’s disease. http://www.aofas.org. Accessed on 10/21/2017. -
Question 7 of 44
7. Question
A 22 year-old female presents to the emergency department with left knee pain after a motor vehicle accident. Her left knee struck the dashboard during the accident and now hurts with any movement. On exam she has tenderness to palpation and ecchymosis over the left patella. She has no deformity to the patella and is able to straight leg raise. Active knee extension increases her pain. AP and lateral x-rays of the left knee show a non-displaced transverse patella fracture. What is the next best step in treatment?
Correct
Patella fractures most commonly occur from a direct impact injury such as a fall or motor vehicle accident. Patients who sustain a patella fracture from a dashboard injury must be examined for an ipsilateral hip injury to rule out dislocation or fracture. The extensor mechanism of the knee may be tested by putting a bump under the patient’s knee and having them place the knee in full extension against gravity. Inability to straight leg raise may indicate an extensor mechanism tear, or tear of the quadriceps tendon and/or the medial and lateral retinaculum. AP and lateral x-rays are routinely ordered in the evaluation of patella fractures. A patella baja “low riding patella” or a patella alta “high riding patella” seen on lateral x-ray may indicate a quadriceps and patella tendon rupture, respectively. 1,2
Most non-displaced patella fractures have a transverse fracture pattern. With non-displaced fractures, the articular cartilage on the undersurface of the patella and the extensor mechanism usually remain intact so further imaging with CT or MRI is generally not necessary. Activities such as ambulation, walking up stairs, and bending down place displacing forces through the knee and should be avoided in patients with a patella fracture. Because the extensor mechanism is not activated in full knee extension, displacing forces can be eliminated with a knee immobilizer. The knee immobilizer locks the knee in extension and prevents quadriceps activation during weight bearing. Therefore the treatment for non-displaced patella fractures includes weight bearing as tolerated with a knee immobilizer on at all times. 1,2References
1. Morris, RM. Fractures of the Patella and Injuries to the Extensor Mechanism. Pages 1965- 1980
2. Patella fractures. http://www.orthobullets.com. Accessed on 10/18/2017.Incorrect
Patella fractures most commonly occur from a direct impact injury such as a fall or motor vehicle accident. Patients who sustain a patella fracture from a dashboard injury must be examined for an ipsilateral hip injury to rule out dislocation or fracture. The extensor mechanism of the knee may be tested by putting a bump under the patient’s knee and having them place the knee in full extension against gravity. Inability to straight leg raise may indicate an extensor mechanism tear, or tear of the quadriceps tendon and/or the medial and lateral retinaculum. AP and lateral x-rays are routinely ordered in the evaluation of patella fractures. A patella baja “low riding patella” or a patella alta “high riding patella” seen on lateral x-ray may indicate a quadriceps and patella tendon rupture, respectively. 1,2
Most non-displaced patella fractures have a transverse fracture pattern. With non-displaced fractures, the articular cartilage on the undersurface of the patella and the extensor mechanism usually remain intact so further imaging with CT or MRI is generally not necessary. Activities such as ambulation, walking up stairs, and bending down place displacing forces through the knee and should be avoided in patients with a patella fracture. Because the extensor mechanism is not activated in full knee extension, displacing forces can be eliminated with a knee immobilizer. The knee immobilizer locks the knee in extension and prevents quadriceps activation during weight bearing. Therefore the treatment for non-displaced patella fractures includes weight bearing as tolerated with a knee immobilizer on at all times. 1,2References
1. Morris, RM. Fractures of the Patella and Injuries to the Extensor Mechanism. Pages 1965- 1980
2. Patella fractures. http://www.orthobullets.com. Accessed on 10/18/2017. -
Question 8 of 44
8. Question
A 49 year-old female presents with a 4 month history of left shoulder pain. She enjoys going to the gym and cross training but her shoulder has been bothering her with overhead activities. She received a subacromial injection 2 months ago then did several weeks of physical therapy with minimal relief. AP and outlet radiographs show no abnormalities. MRI shows a partial thickness articular sided rotator cuff tear of the left shoulder. The patient is taken to the operating for a left shoulder arthroscopy which reveals a partial tear through 60% of the depth of the supraspinatus tendon. What would be the next best step in treatment?
Correct
Partial thickness tears of the rotator cuff present a treatment dilemma for clinicians who treat shoulder pain. These patients often fail conservative treatment with rest, NSAIDS, subacromial injections, and physical therapy before electing for surgery. Partial thickness tears have limited healing potential and will likely progress over time. Despite this, most patients will continue to improve with conservative treatment up to 18 months. However, it is reasonable to consider surgery when conservative treatment has failed for a period of 3-6 months. Many partial tears that are treated with arthroscopy, rotator cuff debridement, and acromioplasty have good to excellent results at 2 to 5 years. However, several factors must be considered during shoulder arthroscopically that help the surgeon determine the best course of treatment. Such factors include location of the partial rotator cuff tear, size of the tear, and other should pathology seen during arthroscopy. Arthroscopic debridement of partial tears is essential in determining the true depth of the tear and may stimulate healing and remove inflammatory cells. Partial thickness tears can occur on the articular side, intratendinous, bursal side, or combination thereof. Articular sided supraspinatus tendon tears are the most common location in older patients and bursal sided tears are typically more symptomatic. Bursal sided tears are more likely to progress to symptomatic full thickness tears than articular sided tears. Partial tears greater than 50% of the tendon depth usually do poorly with conservative treatment. The thickness of a rotator cuff measures approximately 11-12 mm so a tear >6 mm in depth on the articular side and >3 mm in depth on the bursal side will do poorly with observation and should be treated with a rotator cuff repair. When the partial tear is >50% of the tendon depth the tear is often completed then repaired back down to the humerus with suture anchors. Younger overhead athletes with partial rotator cuff tears are the exception to these rules as non-surgical treatment should be the mainstay of treatment. These patients often fail to return to the same level of activity after rotator cuff surgery, particularly repair. Adequate rehab, rest, NSAIDS, and possible steroid injections can effectively resolve symptoms in this population. 1,2
References:
1. Wolff AB, Sethi P, Sutton KM, Covey AS, Magit DP, Medvecky M. Partial Thickness Rotator Cuff Tears. JAAOS 2006; 14: 715-725.
2. Shaffer B, Huttman D. Rotator Cuff Tears in the Throwing Athlete. 2014. Sports Med Arthrosc Rev. Volume 22, number 2. 101-109.Incorrect
Partial thickness tears of the rotator cuff present a treatment dilemma for clinicians who treat shoulder pain. These patients often fail conservative treatment with rest, NSAIDS, subacromial injections, and physical therapy before electing for surgery. Partial thickness tears have limited healing potential and will likely progress over time. Despite this, most patients will continue to improve with conservative treatment up to 18 months. However, it is reasonable to consider surgery when conservative treatment has failed for a period of 3-6 months. Many partial tears that are treated with arthroscopy, rotator cuff debridement, and acromioplasty have good to excellent results at 2 to 5 years. However, several factors must be considered during shoulder arthroscopically that help the surgeon determine the best course of treatment. Such factors include location of the partial rotator cuff tear, size of the tear, and other should pathology seen during arthroscopy. Arthroscopic debridement of partial tears is essential in determining the true depth of the tear and may stimulate healing and remove inflammatory cells. Partial thickness tears can occur on the articular side, intratendinous, bursal side, or combination thereof. Articular sided supraspinatus tendon tears are the most common location in older patients and bursal sided tears are typically more symptomatic. Bursal sided tears are more likely to progress to symptomatic full thickness tears than articular sided tears. Partial tears greater than 50% of the tendon depth usually do poorly with conservative treatment. The thickness of a rotator cuff measures approximately 11-12 mm so a tear >6 mm in depth on the articular side and >3 mm in depth on the bursal side will do poorly with observation and should be treated with a rotator cuff repair. When the partial tear is >50% of the tendon depth the tear is often completed then repaired back down to the humerus with suture anchors. Younger overhead athletes with partial rotator cuff tears are the exception to these rules as non-surgical treatment should be the mainstay of treatment. These patients often fail to return to the same level of activity after rotator cuff surgery, particularly repair. Adequate rehab, rest, NSAIDS, and possible steroid injections can effectively resolve symptoms in this population. 1,2
References:
1. Wolff AB, Sethi P, Sutton KM, Covey AS, Magit DP, Medvecky M. Partial Thickness Rotator Cuff Tears. JAAOS 2006; 14: 715-725.
2. Shaffer B, Huttman D. Rotator Cuff Tears in the Throwing Athlete. 2014. Sports Med Arthrosc Rev. Volume 22, number 2. 101-109. -
Question 9 of 44
9. Question
A 48 year-old male presents to the emergency department by way of ambulance after a motorcycle accident. He was thrown off his bike after hitting another vehicle and sustained a left open femur fracture. He was wearing a helmet and denies loss of consciousness. On exam you note a Gustilo type IIIA open femur fracture shown in figure 1. AP and lateral trauma x-rays are shown in figures 2 and 3. After orthopedic consultation the patient is scheduled for an urgent irrigation and debridement. In terms of preventing a deep infection postoperatively, which choice has the least influence on this outcome?
Correct
The treatment goal of any open fracture is to make sure the wound is cleaned early, the fracture is stabilized, and all measures are taken to prevent a deep infection postoperatively. Complications of an open fracture may include superficial wound infection, nonunion of the fracture, deep soft tissue infection, and osteomyelitis. Factors such as early irrigation and debridement, early antibiotic administration, fracture severity, neurovascular injury, obesity, and diabetes have all been shown to influence the rate of a postoperative deep infection. Time to surgery, however, may be the least important of these factors. Emergent debridement within 6 hours has long been used as a general guideline for all open fractures. However, time to operative debridement can be influenced by a variety of factors including time to emergency room arrival, patient’s medical status in regards to operating room clearance, and availability of the operating room team. Orthopedic surgeons are often unable to bring patients with open fractures to the operating room within 6 hours due to these factors. The general belief that I&D must occur within 6 hours of injury in order to reduce the risk of postoperative infection has been proven untrue in numerous studies. Weber et al. published a large prospective cohort study of almost 800 open long bone fractures and concluded the development of deep infection was not associated with time to initial surgery. Weber et al and other studies have shown that factors such as injury severity (increasing Gustilo type fracture), late antibiotic administration, neurovascular injury, obesity, and diabetes do increase the rate of deep infection postoperatively. These studies have changed the way many open fractures are treated. Often times these fractures were taken to the OR in the middle of the night under suboptimal conditions for the surgeon and surgical team to meet the 6 hour window. These studies confirm that emergent I&D within 24 hours is as safe and effective as urgent I&D within 6 hours. I&D done within 24 hours, and during day time hours when all operative resources are readily available to the surgeon, is appropriate and preferable. High energy open long bone fractures with evidence of wound contamination generally require multiple I&D’s prior to wound closure. Common protocols call for I&D, obtain surgical site cultures, and fracture stabilization within 24 hours of injury. Patients are usually taken back for repeat I&D at 48 hours and cultures are repeated if the initial cultures were positive. Definitive fixation and wound closure can be performed if operative cultures come back negative, there is adequate soft tissue coverage for closure, and if the surgeon deems appropriate. 1,2
References
1. Werner CM, Pierpont Y, Pollak AN. The Urgency of Surgical Debridement in the Management of Open Fractures. JAAOS 2008; 16: 369-375.
2. Weber D, Dulai SK, Bergman J, Buckley R, Beaupre LA. Time to Initial Operative Treatment Following Open Fracture Does Not Impact Development of Deep Infection: A Prospective Cohort Study of 736 Subjects. Journal of Orthopaedic Trauma: November 2014 – Volume 28 – Issue 11 – p 613–619.Incorrect
The treatment goal of any open fracture is to make sure the wound is cleaned early, the fracture is stabilized, and all measures are taken to prevent a deep infection postoperatively. Complications of an open fracture may include superficial wound infection, nonunion of the fracture, deep soft tissue infection, and osteomyelitis. Factors such as early irrigation and debridement, early antibiotic administration, fracture severity, neurovascular injury, obesity, and diabetes have all been shown to influence the rate of a postoperative deep infection. Time to surgery, however, may be the least important of these factors. Emergent debridement within 6 hours has long been used as a general guideline for all open fractures. However, time to operative debridement can be influenced by a variety of factors including time to emergency room arrival, patient’s medical status in regards to operating room clearance, and availability of the operating room team. Orthopedic surgeons are often unable to bring patients with open fractures to the operating room within 6 hours due to these factors. The general belief that I&D must occur within 6 hours of injury in order to reduce the risk of postoperative infection has been proven untrue in numerous studies. Weber et al. published a large prospective cohort study of almost 800 open long bone fractures and concluded the development of deep infection was not associated with time to initial surgery. Weber et al and other studies have shown that factors such as injury severity (increasing Gustilo type fracture), late antibiotic administration, neurovascular injury, obesity, and diabetes do increase the rate of deep infection postoperatively. These studies have changed the way many open fractures are treated. Often times these fractures were taken to the OR in the middle of the night under suboptimal conditions for the surgeon and surgical team to meet the 6 hour window. These studies confirm that emergent I&D within 24 hours is as safe and effective as urgent I&D within 6 hours. I&D done within 24 hours, and during day time hours when all operative resources are readily available to the surgeon, is appropriate and preferable. High energy open long bone fractures with evidence of wound contamination generally require multiple I&D’s prior to wound closure. Common protocols call for I&D, obtain surgical site cultures, and fracture stabilization within 24 hours of injury. Patients are usually taken back for repeat I&D at 48 hours and cultures are repeated if the initial cultures were positive. Definitive fixation and wound closure can be performed if operative cultures come back negative, there is adequate soft tissue coverage for closure, and if the surgeon deems appropriate. 1,2
References
1. Werner CM, Pierpont Y, Pollak AN. The Urgency of Surgical Debridement in the Management of Open Fractures. JAAOS 2008; 16: 369-375.
2. Weber D, Dulai SK, Bergman J, Buckley R, Beaupre LA. Time to Initial Operative Treatment Following Open Fracture Does Not Impact Development of Deep Infection: A Prospective Cohort Study of 736 Subjects. Journal of Orthopaedic Trauma: November 2014 – Volume 28 – Issue 11 – p 613–619. -
Question 10 of 44
10. Question
A 17-year old male presents to your clinic after sustaining a right knee injury during a lacrosse game. He felt a “pop” in the knee as he pivoted to avoid an opponent. His primary care physician ordered an MRI after the injury which showed a complete anterior cruciate ligament rupture. You discuss with the patient the recommendations of ACL reconstruction including which graft types are available. For this patient, which would be the recommended graft choice to reconstruct his ACL?
Correct
The anterior cruciate ligament (ACL) is a crucial stabilizing structure in the knee that primarily provides restraint to anterior translation and rotation of the tibia. A deficient ACL causes the knee to give out or become unstable with activities that require running and cutting such as athletics. A deficient ACL may also cause buckling of the knee with day to day activities such as walking down stairs or changing direction quickly. In most active patients an ACL deficient knee must be treated acutely as damage to the menisci and articular cartilage can occur with instability episodes. This involves an arthroscopic procedure that takes out the torn tissue and reconstructs the ligament with a graft, either from the patient’s own tissue (autograft) or donated cadaver tissue (allograft).
There are several factors to be considered as patients and orthopedic surgeons choose which graft to use. The differences of the success rates between graft choices are small so the benefits and downfalls of each should be discussed and considered. Autografts have the advantage of using the patient’s own tissue which tend to heal a little faster than donated tissue. The most commonly used graft type in the US is a bone patella bone autograft with the second type being hamstring autograft. Professional athletes most commonly choose bone patella bone autograft followed by hamstring autograft. The downside of autografts is the additional pain postoperatively that is associated with harvesting the graft. Allografts have a higher failure rate in young athletes but have less pain after surgery and tend to rehab a little quicker. This is why recreational athletes over 35 to 40 years of age generally choose an allograft for a quicker return to work. Surgeon’s preference also plays a major role when patients select graft choice. Graft choice can come with its own unique surgical approach/technique and often surgeons choose the graft and technique they are most familiar with.References
Furio N, Yin KL, Marx RG. Graft Selection and Preparation in Anterior Cruciate Ligament Reconstruction. JBJS Journal of Orthopaedics for Physician Assistants: January – March 2017 – Volume 5 – Issue 1 – p e6
ACL Tear. http://www.orthobullets.com. Accessed on 4/24/2017.Incorrect
The anterior cruciate ligament (ACL) is a crucial stabilizing structure in the knee that primarily provides restraint to anterior translation and rotation of the tibia. A deficient ACL causes the knee to give out or become unstable with activities that require running and cutting such as athletics. A deficient ACL may also cause buckling of the knee with day to day activities such as walking down stairs or changing direction quickly. In most active patients an ACL deficient knee must be treated acutely as damage to the menisci and articular cartilage can occur with instability episodes. This involves an arthroscopic procedure that takes out the torn tissue and reconstructs the ligament with a graft, either from the patient’s own tissue (autograft) or donated cadaver tissue (allograft).
There are several factors to be considered as patients and orthopedic surgeons choose which graft to use. The differences of the success rates between graft choices are small so the benefits and downfalls of each should be discussed and considered. Autografts have the advantage of using the patient’s own tissue which tend to heal a little faster than donated tissue. The most commonly used graft type in the US is a bone patella bone autograft with the second type being hamstring autograft. Professional athletes most commonly choose bone patella bone autograft followed by hamstring autograft. The downside of autografts is the additional pain postoperatively that is associated with harvesting the graft. Allografts have a higher failure rate in young athletes but have less pain after surgery and tend to rehab a little quicker. This is why recreational athletes over 35 to 40 years of age generally choose an allograft for a quicker return to work. Surgeon’s preference also plays a major role when patients select graft choice. Graft choice can come with its own unique surgical approach/technique and often surgeons choose the graft and technique they are most familiar with.References
Furio N, Yin KL, Marx RG. Graft Selection and Preparation in Anterior Cruciate Ligament Reconstruction. JBJS Journal of Orthopaedics for Physician Assistants: January – March 2017 – Volume 5 – Issue 1 – p e6
ACL Tear. http://www.orthobullets.com. Accessed on 4/24/2017. -
Question 11 of 44
11. Question
A 25 year-old male presents to the emergency department after a fall while rock climbing. He landed on his right ankle as he fell 10 feet off the face of the rock wall. He is unable to bear weight on the right lower extremity. Lateral and oblique x-rays (figures 1 and 2) show a minimally displaced talar neck fracture. What is the next best step in this patient’s treatment?
Correct
Fractures of the talus are difficult to treat due to their high complication rates and potential for long term disability. The talus is 60% covered with articular cartilage which limits the available area for vascular perfusion. As a result, displaced fractures can disrupt blood supply to the talus leading to a higher risk for osteonecrosis. Displaced fractures also disrupt the congruity of the articular cartilage which can result in post-traumatic arthritis. The most common fracture types are chip or avulsion fractures, although fractures through the talar neck, body, head, lateral process, and posterior process can occur. Fractures of the neck, body, and head are usually found on x-ray, however subtle fractures which frequently occur in the lateral process often require CT to diagnose. 1,2
Most talus fractures require operative fixation. Fractures that appear non-displaced on x-ray should have a CT to confirm joint congruity. CT also assesses for malalignment as the talus typically displaces in a plantar flexed and varus position. If the fracture is non-displaced on CT than a short leg weight bearing cast can be applied for 6-8 weeks, followed by a weight bearing cast or boot for an additional 4-6 weeks. If a step off or displacement is noted on CT than the patient should have an open reduction and internal fixation. 1,2
References
1. Sanders, DW. Fractures of the Talus. Rockwood and Green’s Fractures in Adults, 6th Edition. Lippincott Williams & Wilkins. Philadelphia, PA 2006. Pages 2249-2289
2. Fractures of the Talar Neck. http://www.orthobullets.com. Accessed on 1/16/2018.Incorrect
Fractures of the talus are difficult to treat due to their high complication rates and potential for long term disability. The talus is 60% covered with articular cartilage which limits the available area for vascular perfusion. As a result, displaced fractures can disrupt blood supply to the talus leading to a higher risk for osteonecrosis. Displaced fractures also disrupt the congruity of the articular cartilage which can result in post-traumatic arthritis. The most common fracture types are chip or avulsion fractures, although fractures through the talar neck, body, head, lateral process, and posterior process can occur. Fractures of the neck, body, and head are usually found on x-ray, however subtle fractures which frequently occur in the lateral process often require CT to diagnose. 1,2
Most talus fractures require operative fixation. Fractures that appear non-displaced on x-ray should have a CT to confirm joint congruity. CT also assesses for malalignment as the talus typically displaces in a plantar flexed and varus position. If the fracture is non-displaced on CT than a short leg weight bearing cast can be applied for 6-8 weeks, followed by a weight bearing cast or boot for an additional 4-6 weeks. If a step off or displacement is noted on CT than the patient should have an open reduction and internal fixation. 1,2
References
1. Sanders, DW. Fractures of the Talus. Rockwood and Green’s Fractures in Adults, 6th Edition. Lippincott Williams & Wilkins. Philadelphia, PA 2006. Pages 2249-2289
2. Fractures of the Talar Neck. http://www.orthobullets.com. Accessed on 1/16/2018. -
Question 12 of 44
12. Question
A 48 year-old female presents to the office with left foot pain from a fall two days prior. She complains of pain with weight bearing and noticeable bruising over her midfoot. AP and lateral x-rays (figures 1 and 2) show a medial cuneiform fracture with mild displacement. What is the most appropriate treatment option for this patient?
Correct
Fractures of the midfoot are a relatively uncommon injury. The midfoot consists of five bones including the navicular, cuboid, and the medial, middle, and lateral cuneiform. The bones of the midfoot are tightly bound together by strong dense plantar ligaments and are relatively immobile. Together the bones and ligaments help maintain the normal arch of the foot during weight bearing. Injuries or deformity to the midfoot can significantly affect both hindfoot and forefoot alignment. 1,2
The medial, middle, and lateral cuneiform bones align side by side on the medial foot and provide rigid support to the medial longitudinal arch during weight bearing activities. Cuneiform injuries are rare but can occur with an axial load and in association with tarsometatarsal joint injuries. X-ray with three views of the foot should be obtained if a fracture to the cuneiform bones is suspected. Swelling, pain to palpation over the midfoot, or pain with weight bearing are indications for x-ray. If x-rays are negative with a suspected midfoot injury than single leg weight bearing (stress view) x-ray should be obtained if possible to determine if instability is present. Disruption of the intercuneiform ligaments can occur and cause the cuneiform bones to spread apart with weight bearing (instability). CT is the most common follow-up study used if a fracture or instability is suspected but not apparent on x-ray. Displaced and unstable injuries of the cuneiform bones are treated with open reduction and internal fixation. Most cuneiform fractures are stable and non-displaced or minimally displaced fractures can be treated in a non-weight bearing boot or cast for 6-8 weeks. Walking on the foot may cause a cuneiform fracture to displace so weight bearing is generally not recommend until the fracture heals radiographically.References
1. Tarsal navicular fractures. http://www.orthobullets.com. Accessed on 9/27/2017.
2. Early JS. Fractures and dislocations of the Midfoot and Forefoot. Rockwood and Wilkins’ Fractures in Adults. 6th ed. Philadelphia, PA: Lippincott Williams & Wilkins; 2006.Incorrect
Fractures of the midfoot are a relatively uncommon injury. The midfoot consists of five bones including the navicular, cuboid, and the medial, middle, and lateral cuneiform. The bones of the midfoot are tightly bound together by strong dense plantar ligaments and are relatively immobile. Together the bones and ligaments help maintain the normal arch of the foot during weight bearing. Injuries or deformity to the midfoot can significantly affect both hindfoot and forefoot alignment. 1,2
The medial, middle, and lateral cuneiform bones align side by side on the medial foot and provide rigid support to the medial longitudinal arch during weight bearing activities. Cuneiform injuries are rare but can occur with an axial load and in association with tarsometatarsal joint injuries. X-ray with three views of the foot should be obtained if a fracture to the cuneiform bones is suspected. Swelling, pain to palpation over the midfoot, or pain with weight bearing are indications for x-ray. If x-rays are negative with a suspected midfoot injury than single leg weight bearing (stress view) x-ray should be obtained if possible to determine if instability is present. Disruption of the intercuneiform ligaments can occur and cause the cuneiform bones to spread apart with weight bearing (instability). CT is the most common follow-up study used if a fracture or instability is suspected but not apparent on x-ray. Displaced and unstable injuries of the cuneiform bones are treated with open reduction and internal fixation. Most cuneiform fractures are stable and non-displaced or minimally displaced fractures can be treated in a non-weight bearing boot or cast for 6-8 weeks. Walking on the foot may cause a cuneiform fracture to displace so weight bearing is generally not recommend until the fracture heals radiographically.References
1. Tarsal navicular fractures. http://www.orthobullets.com. Accessed on 9/27/2017.
2. Early JS. Fractures and dislocations of the Midfoot and Forefoot. Rockwood and Wilkins’ Fractures in Adults. 6th ed. Philadelphia, PA: Lippincott Williams & Wilkins; 2006. -
Question 13 of 44
13. Question
A 7 year-old male presents to emergency department with a left arm injury that occurred a few hours earlier. He was riding on a scooter and fell forward landing on his outstretched hand. On exam you note a slight angular deformity to the left forearm. AP and lateral x-rays show a distal 1/3 radius fracture with plastic deformation (figures 1 and 2). Which choice below is true regarding pediatric bone and plastic deformation?
Correct
Plastic deformation, or traumatic bowing, is an irreversible change in the shape of bone after an injury which occurs almost exclusively in pediatric forearm fractures. Plastic deformation occurs when a large amount of force is placed on bone and the bone stays in the deformed position (plasticity) even after the force is removed. The modulus of elasticity, or the measure of stiffness of a solid material, is higher in adult bone than pediatric bone. (Another example of modulus of elasticity: metal implants used in orthopedic surgery have a higher modulus of elasticity than bone). Mineral content of bone plays a large role in its elastic properties. Pediatric corticol bone has lower mineral content than adult bone and therefore has weaker bending strength. With aging, the increased mineralization stiffens the collagen and hydroxyapatite in bone, which decreases the amount of deformity the bone will tolerate before fracture. Unlike adult bone, pediatric bone is able to absorb significant bending forces due to plastic deformation properties prior to fracturing. 1,2
A plastic deformity of the forearm can limit forearm supination and pronation as the deformed radius and ulna encroach upon the interosseous space. In general, plastic deformities are thought to have less remodeling potential and can be more difficult to reduce than other angulated pediatric forearm fractures. Therefore the treatment approach is unique as the threshold for closed reduction under anesthesia is less. One recommendation includes close reducing plastic deformities under general anesthesia with greater than 10 degrees of angulation in all patients over 6 years of age. In general, displaced forearm fractures (non-plastic deformity) generally tolerate 15-20 degrees of angulation before closed reduction is attempted in patients less than 10 years of age. 1,2References
1. Mabrey JD, Fitch RD. Deformation in pediatric fractures: mechanism and treatment. Pediatr Orthop. 1989 May-Jun;9(3):310-4.
2. Wall EJ, Mehlman CT. Injuries to the Shafts of the Radius and Ulna. Rockwood and Wilkins’ Fractures in Children 6th Edition. 2001 Philadelpia, PA. Pg 417-418Incorrect
Plastic deformation, or traumatic bowing, is an irreversible change in the shape of bone after an injury which occurs almost exclusively in pediatric forearm fractures. Plastic deformation occurs when a large amount of force is placed on bone and the bone stays in the deformed position (plasticity) even after the force is removed. The modulus of elasticity, or the measure of stiffness of a solid material, is higher in adult bone than pediatric bone. (Another example of modulus of elasticity: metal implants used in orthopedic surgery have a higher modulus of elasticity than bone). Mineral content of bone plays a large role in its elastic properties. Pediatric corticol bone has lower mineral content than adult bone and therefore has weaker bending strength. With aging, the increased mineralization stiffens the collagen and hydroxyapatite in bone, which decreases the amount of deformity the bone will tolerate before fracture. Unlike adult bone, pediatric bone is able to absorb significant bending forces due to plastic deformation properties prior to fracturing. 1,2
A plastic deformity of the forearm can limit forearm supination and pronation as the deformed radius and ulna encroach upon the interosseous space. In general, plastic deformities are thought to have less remodeling potential and can be more difficult to reduce than other angulated pediatric forearm fractures. Therefore the treatment approach is unique as the threshold for closed reduction under anesthesia is less. One recommendation includes close reducing plastic deformities under general anesthesia with greater than 10 degrees of angulation in all patients over 6 years of age. In general, displaced forearm fractures (non-plastic deformity) generally tolerate 15-20 degrees of angulation before closed reduction is attempted in patients less than 10 years of age. 1,2References
1. Mabrey JD, Fitch RD. Deformation in pediatric fractures: mechanism and treatment. Pediatr Orthop. 1989 May-Jun;9(3):310-4.
2. Wall EJ, Mehlman CT. Injuries to the Shafts of the Radius and Ulna. Rockwood and Wilkins’ Fractures in Children 6th Edition. 2001 Philadelpia, PA. Pg 417-418 -
Question 14 of 44
14. Question
A 72 year-old male presents to your office with pain and stiffness in his right hand. The pain is worse in his fingers, especially the ring finger. He is an avid golfer and is having difficulty swinging his club due to the pain. AP x-ray of the hand and lateral of the right ring finger (figures 1 and 2) show severe arthritis throughout the proximal interphalangeal (PIP) and distal interphalangeal (DIP) joints. He is looking for surgical treatment options as he has tried years of rest, ice, and NSAIDs. Which choice is a recommended surgical treatment for finger arthritis?
Correct
Arthritis of the fingers is a common condition that can be found in most adults in advanced age, especially in women. Up to 70% of women over 80 years old will have arthritis at the DIP joint and hand arthritis is seen as a 10:1 female to male ratio. Most patients will not be symptomatic despite arthritis on x-ray. In fact, there is very little correlation between the degree of arthritis on x-ray and pain level. Osteoarthritis effects the DIP joints more than the PIP and MCP joints. Symptoms of finger arthritis include intermittent pain and stiffness when using the hands. Arthritic deformities can also be present in the fingers. Heberden’s nodules are osteophytes at the DIP joints and Bouchard’s nodules are osteophytes at the PIP joint. 1,2
The first line treatment for finger arthritis is ice, NSAID’s, and activity modification. Intra-articular injections can be performed for temporary relief, although the injections can be technically difficult and the relief is usually short lived. The two most common surgical options include arthroplasty (replacement) or arthrodesis (fusion). DIP joint arthritis is generally treated with an arthrodesis. The bone stock of the distal phalanx is limited and doesn’t support an implant required for arthroplasty. Arthroplasty with a silicone implant is the preferred surgical treatment for PIP arthritis. Arthroplasty better preserves joint motion and function compared to arthrodesis. 2
References
1. Finger Arthritis. http://www.orthobullets.com. Accessed on 1/18/2018
2. Beldner S, Polatsch DB. Arthrodesis of the metacarpophalangeal joint and interphalangeal joints of the hand: current concepts. JAAOS 2016; 24: 290-297.Incorrect
Arthritis of the fingers is a common condition that can be found in most adults in advanced age, especially in women. Up to 70% of women over 80 years old will have arthritis at the DIP joint and hand arthritis is seen as a 10:1 female to male ratio. Most patients will not be symptomatic despite arthritis on x-ray. In fact, there is very little correlation between the degree of arthritis on x-ray and pain level. Osteoarthritis effects the DIP joints more than the PIP and MCP joints. Symptoms of finger arthritis include intermittent pain and stiffness when using the hands. Arthritic deformities can also be present in the fingers. Heberden’s nodules are osteophytes at the DIP joints and Bouchard’s nodules are osteophytes at the PIP joint. 1,2
The first line treatment for finger arthritis is ice, NSAID’s, and activity modification. Intra-articular injections can be performed for temporary relief, although the injections can be technically difficult and the relief is usually short lived. The two most common surgical options include arthroplasty (replacement) or arthrodesis (fusion). DIP joint arthritis is generally treated with an arthrodesis. The bone stock of the distal phalanx is limited and doesn’t support an implant required for arthroplasty. Arthroplasty with a silicone implant is the preferred surgical treatment for PIP arthritis. Arthroplasty better preserves joint motion and function compared to arthrodesis. 2
References
1. Finger Arthritis. http://www.orthobullets.com. Accessed on 1/18/2018
2. Beldner S, Polatsch DB. Arthrodesis of the metacarpophalangeal joint and interphalangeal joints of the hand: current concepts. JAAOS 2016; 24: 290-297. -
Question 15 of 44
15. Question
A 55 year-old male presents to the ED with right foot pain after falling from a ladder. X-rays show displaced 2nd, 3rd, and 4th metatarsal fractures and dislocations of the 4th and 5th metatarsophalangeal joints (figures 1 and 2). What is the best initial treatment option for this patient?Correct
Fracture dislocations of the metatarsophalangeal (MTP) joint are rare injuries. Management of these injuries in the emergency room setting includes an attempt at closed reduction using local anesthesia. Any deviation of the metatarsal heads should be reduced to restore alignment. Closed reduction is successful around 50% of the time with MTP joint dislocations. Multiple metatarsal neck fractures can be very difficult to reduce. Whether the reduction is successful or not, the patient can follow-up with orthopedics in 24-48 hours for definitive fixation. An irreducible fracture dislocation of the MTP is not a surgical emergency and can be treated in an outpatient setting. Indications for closed reduction include fracture angulation > 10 degrees along the long axis of the metatarsal or > 4 mm of translation of the shaft. The goal of closed reduction is to reduce a plantar flexed metatarsal head and impale the head back on the shaft. A percutaneous pin is usually placed through the base of the proximal phalanx, though the metatarsal head and fracture, and into the metatarsal shaft to secure reduction. Fixation helps prevent the natural tendency of the distal metatarsal fractures to drift laterally. Post-operatively the patient is allowed to weight bear through the heel. Pins are often removed at 4 weeks post-operatively. 1,2
Simple dislocations, such as the 5th MTP joint in this patient, can be treated successfully with closed reduction alone. The surrounding soft tissues of the MTP joint provide adequate stability without the need for pin fixation. The main supporting structures include medial and lateral MTP joint collateral ligaments and the plantar plate. The plantar plate is a broad, thick ligamentous structure that spans the plantar aspect of the MTP joint. Open treatment can often disrupt these soft tissue structures causing MTP joint instability so closed treatment is preferred. 1,2References
1. Early JS. Fractures and Dislocations of the Midfoot and Forefoot. Rockwood and Green Fractures in Adults, 6th Edition. Lippincott Williams and Wilkins. Philadelphia, PA 2006. 2370-2381.
2. Aitken, AP; Poulson, D: Dislocations of the tarsometarsal joint. J. Bone Joint Surg., 45A:246–260, 1963.Incorrect
Fracture dislocations of the metatarsophalangeal (MTP) joint are rare injuries. Management of these injuries in the emergency room setting includes an attempt at closed reduction using local anesthesia. Any deviation of the metatarsal heads should be reduced to restore alignment. Closed reduction is successful around 50% of the time with MTP joint dislocations. Multiple metatarsal neck fractures can be very difficult to reduce. Whether the reduction is successful or not, the patient can follow-up with orthopedics in 24-48 hours for definitive fixation. An irreducible fracture dislocation of the MTP is not a surgical emergency and can be treated in an outpatient setting. Indications for closed reduction include fracture angulation > 10 degrees along the long axis of the metatarsal or > 4 mm of translation of the shaft. The goal of closed reduction is to reduce a plantar flexed metatarsal head and impale the head back on the shaft. A percutaneous pin is usually placed through the base of the proximal phalanx, though the metatarsal head and fracture, and into the metatarsal shaft to secure reduction. Fixation helps prevent the natural tendency of the distal metatarsal fractures to drift laterally. Post-operatively the patient is allowed to weight bear through the heel. Pins are often removed at 4 weeks post-operatively. 1,2
Simple dislocations, such as the 5th MTP joint in this patient, can be treated successfully with closed reduction alone. The surrounding soft tissues of the MTP joint provide adequate stability without the need for pin fixation. The main supporting structures include medial and lateral MTP joint collateral ligaments and the plantar plate. The plantar plate is a broad, thick ligamentous structure that spans the plantar aspect of the MTP joint. Open treatment can often disrupt these soft tissue structures causing MTP joint instability so closed treatment is preferred. 1,2References
1. Early JS. Fractures and Dislocations of the Midfoot and Forefoot. Rockwood and Green Fractures in Adults, 6th Edition. Lippincott Williams and Wilkins. Philadelphia, PA 2006. 2370-2381.
2. Aitken, AP; Poulson, D: Dislocations of the tarsometarsal joint. J. Bone Joint Surg., 45A:246–260, 1963. -
Question 16 of 44
16. Question
A 24 year-old male presents to your office with right wrist pain following a motorcycle accident 2 days ago. A scaphoid view x-ray taken in the emergency department the day of injury shows a non-displaced scaphoid waist fracture (figure 1). Which statement is true regarding scaphoid fractures?
Correct
Scaphoid fractures are the most common of all carpal fractures. Injury is often the result of a fall on an outstretched hand with the wrist in extension. Most (80%) of the blood supply to the scaphoid is supplied via branches of the radial artery that start distally and perfuse in a retrograde manner (distal to proximal). Fractures through the scaphoid can interrupt this tenuous blood supply and are prone to slow healing. Because the blood supply to the scaphoid is decreased proximally, fractures are less likely to heal as they occur more proximally. Proximal scaphoid fractures have the highest rate of nonunion. The waist is the most common location for scaphoid fractures to occur, followed by the proximal third, then the distal third. 1
In general non-displaced, or up to 1mm displaced, scaphoid fractures can be treated successfully without surgery. Scaphoid waist fractures are placed in a thumb spica cast for 2-3 months or until the fracture is healed. Healing rates of 90% with cast treatment are equal to the healing rates with surgical treatment and without the increased complication rate of surgery. Indications for surgery include proximal pole and displaced fractures. Without early recognition and treatment of scaphoid fractures patients can go on to develop avascular necrosis of the scaphoid, scapholunate advanced collapse, and wrist arthritis. Any suspected scaphoid fracture with normal x-rays should be either followed closely or evaluated with an MRI to avoid these potential complications. 1,2References
1. Scaphoid Fractures. http://www.orthobullets.com. Accessed on 7/22/17.
2. Ibrahim T, Qureshi A, Sutton AJ, Dias JJ. Surgical Versus Nonsurgical Treatment of Acute Minimally Displaced and Undisplaced Scaphoid Waist Fractures: Pairwise and Network Meta-Analyses of Randomized Controlled Trials. The Journal of Hand Surgery. Volume 36, Issue 11, November 2011, Pages 1759-1768.Incorrect
Scaphoid fractures are the most common of all carpal fractures. Injury is often the result of a fall on an outstretched hand with the wrist in extension. Most (80%) of the blood supply to the scaphoid is supplied via branches of the radial artery that start distally and perfuse in a retrograde manner (distal to proximal). Fractures through the scaphoid can interrupt this tenuous blood supply and are prone to slow healing. Because the blood supply to the scaphoid is decreased proximally, fractures are less likely to heal as they occur more proximally. Proximal scaphoid fractures have the highest rate of nonunion. The waist is the most common location for scaphoid fractures to occur, followed by the proximal third, then the distal third. 1
In general non-displaced, or up to 1mm displaced, scaphoid fractures can be treated successfully without surgery. Scaphoid waist fractures are placed in a thumb spica cast for 2-3 months or until the fracture is healed. Healing rates of 90% with cast treatment are equal to the healing rates with surgical treatment and without the increased complication rate of surgery. Indications for surgery include proximal pole and displaced fractures. Without early recognition and treatment of scaphoid fractures patients can go on to develop avascular necrosis of the scaphoid, scapholunate advanced collapse, and wrist arthritis. Any suspected scaphoid fracture with normal x-rays should be either followed closely or evaluated with an MRI to avoid these potential complications. 1,2References
1. Scaphoid Fractures. http://www.orthobullets.com. Accessed on 7/22/17.
2. Ibrahim T, Qureshi A, Sutton AJ, Dias JJ. Surgical Versus Nonsurgical Treatment of Acute Minimally Displaced and Undisplaced Scaphoid Waist Fractures: Pairwise and Network Meta-Analyses of Randomized Controlled Trials. The Journal of Hand Surgery. Volume 36, Issue 11, November 2011, Pages 1759-1768. -
Question 17 of 44
17. Question
An 80 year-old female presents to the office with 3 months of right shin pain. She had a revision total knee replacement 5 years ago for aseptic loosening and had been doing well. Her pain is made worse with weight bearing and relieved with rest. She has tried NSAIDS and 4 weeks of touch down weight bearing with little improvement. On exam she has no effusion and full passive motion without pain. AP and lateral x-rays taken in the office show no abnormalities. A bone scan of the lower extremities shows increase uptake of the right tibial shaft consistent with a tibial stress reaction. What is the most likely diagnosis?
Correct
The most common complications following total knee arthroplasty include aseptic loosening, infection, instability, and periprosthetic fracture. The most common cause of early TKA failure (within two years) is infection while the most common cause of late failure is aseptic loosening. Approximately 40% of TKA failures overall result from aseptic loosening and 27% are due to septic failure. Aseptic prosthetic loosening can be caused by a variety of factors including implant malalignment, malpositioning, instability, and poor bone quality. This patient has a long uncememted revision metal stem and the implant is press fit into the tibial canal. The press fit metal component has no bony ingrowth potential at the implant-bone interface so any excess motion of the stem can cause the surrounding bone to fatigue. Pain made worse with weight bearing is a sign that the metal implant is starting to cause fatigue, or a stress reaction, at the tibial diaphysis. This is confirmed on the patient’s bone scan. In the work-up of aseptic loosening, x-rays are often negative early in the presentation. With normal x-rays, a bone scan is the next best study of choice. A bone scan can be positive up to two years after TKA so results should be correlated with the clinical scenario. This patient has a positive bone scan 5 years out from revision TKA so the changes in the tibial stem can be ruled pathological. The patient’s bone scan shows increase uptake at the implant-bone interface of the tibial stem suggesting aseptic loosening. Unfortunately when stress reactions occur, as did in this patient, conservative treatment with a period of non-weight bearing is often unsuccessful and a revision surgery is generally required. 1,2
References
1. Sharkey PF, Lichstein PM, Shen C, Tokarski AT, Parvizi J. Why are total knee arthroplasties failing today- has anything changed after 10 years? The Journal of Arthroplasty. Volume 29, Issue 9, September 2014, Pages 1774-1778.
2. Lonner JH, Lotke PA. Aseptic Complications After Total Knee Arthroplasty. JAAOS 1999; 7: 311-324.Incorrect
The most common complications following total knee arthroplasty include aseptic loosening, infection, instability, and periprosthetic fracture. The most common cause of early TKA failure (within two years) is infection while the most common cause of late failure is aseptic loosening. Approximately 40% of TKA failures overall result from aseptic loosening and 27% are due to septic failure. Aseptic prosthetic loosening can be caused by a variety of factors including implant malalignment, malpositioning, instability, and poor bone quality. This patient has a long uncememted revision metal stem and the implant is press fit into the tibial canal. The press fit metal component has no bony ingrowth potential at the implant-bone interface so any excess motion of the stem can cause the surrounding bone to fatigue. Pain made worse with weight bearing is a sign that the metal implant is starting to cause fatigue, or a stress reaction, at the tibial diaphysis. This is confirmed on the patient’s bone scan. In the work-up of aseptic loosening, x-rays are often negative early in the presentation. With normal x-rays, a bone scan is the next best study of choice. A bone scan can be positive up to two years after TKA so results should be correlated with the clinical scenario. This patient has a positive bone scan 5 years out from revision TKA so the changes in the tibial stem can be ruled pathological. The patient’s bone scan shows increase uptake at the implant-bone interface of the tibial stem suggesting aseptic loosening. Unfortunately when stress reactions occur, as did in this patient, conservative treatment with a period of non-weight bearing is often unsuccessful and a revision surgery is generally required. 1,2
References
1. Sharkey PF, Lichstein PM, Shen C, Tokarski AT, Parvizi J. Why are total knee arthroplasties failing today- has anything changed after 10 years? The Journal of Arthroplasty. Volume 29, Issue 9, September 2014, Pages 1774-1778.
2. Lonner JH, Lotke PA. Aseptic Complications After Total Knee Arthroplasty. JAAOS 1999; 7: 311-324. -
Question 18 of 44
18. Question
A 38 year-old male presents to emergency department by way of ambulance after sustaining a right leg injury. He was doing land clearing and had a wood log roll on his leg. X-rays taken in the ED show a closed displaced midshaft tibia fracture. He was placed in a posterior long leg splint and admitted for intramedullary nail fixation the next day. However, in the middle of the night the patient began to complain of excruciating calf pain that was increasing despite additional intravenous pain medication given over the last few hours. You examine the patient with the splint off and note firm compartments of the right lower leg and severe pain with passive dorsiflexion of the ankle. You suspect compartment syndrome and decide to measure the lower extremity compartments. When measuring the compartments, which choice below would indicate compartment syndrome and the need for emergent fasciotomy?
Correct
The lower extremity nerves, muscles, and blood vessels are grouped into four compartments surrounded by fascial tissue. The fascia holds the four compartments together including the anterior, lateral, superficial posterior, and deep posterior compartments. The fascial tissue has limited expansion potential as each compartment is wrapped tightly. Traumatic injuries and bleeding can cause the compartments to swell and without room to expand the pressure within the compartments can elevate. When compartment pressures increase to dangerous levels the vascular and nerve supply to the distal leg can be compromised. Without distal perfusion and an adequate blood supplytissue can begin to die in the foot. Compartment syndrome most commonly occurs with tibial shaft fractures, although rarely seen (occurring in an estimated 2% of tibia fractures). Other causes of compartment syndrome may include bleeding disorders, crush injuries, tight dressings or casts, burns, and over exertion. 1,2
It is crucial to recognize compartment syndrome as a surgical emergency. Classic early signs of acute compartment syndrome of the lower extremity include increasing pain out of proportion despite intravenous pain medication and severe pain with passive stretch of the ankle. If the patient’s clinical signs clearly indicate acute compartment syndrome than emergent fasciotomy should be performed. If the diagnosis remains unclear and other treatments have failed (bivalving or removing a cast or splint, elevation, ice, and intravenous pain medication) than the compartments should be measured. The most common location for compartment syndrome is the anterior compartment but all four compartments should be measured. When measuring the compartments, any measurement where the diastolic pressure minus the compartment pressure is < 30 mmHG indicates compartment syndrome. 1,2
References
1. Compartment syndrome. http://www.orthobullets.com. Accessed on 7/22/17.
2. Olsen SA, Glasgow RR. Acute Compartment Syndrome in Lower Extremity Musculoskeletal Trauma. JAAOS 2005; 13: 436-444.Incorrect
The lower extremity nerves, muscles, and blood vessels are grouped into four compartments surrounded by fascial tissue. The fascia holds the four compartments together including the anterior, lateral, superficial posterior, and deep posterior compartments. The fascial tissue has limited expansion potential as each compartment is wrapped tightly. Traumatic injuries and bleeding can cause the compartments to swell and without room to expand the pressure within the compartments can elevate. When compartment pressures increase to dangerous levels the vascular and nerve supply to the distal leg can be compromised. Without distal perfusion and an adequate blood supplytissue can begin to die in the foot. Compartment syndrome most commonly occurs with tibial shaft fractures, although rarely seen (occurring in an estimated 2% of tibia fractures). Other causes of compartment syndrome may include bleeding disorders, crush injuries, tight dressings or casts, burns, and over exertion. 1,2
It is crucial to recognize compartment syndrome as a surgical emergency. Classic early signs of acute compartment syndrome of the lower extremity include increasing pain out of proportion despite intravenous pain medication and severe pain with passive stretch of the ankle. If the patient’s clinical signs clearly indicate acute compartment syndrome than emergent fasciotomy should be performed. If the diagnosis remains unclear and other treatments have failed (bivalving or removing a cast or splint, elevation, ice, and intravenous pain medication) than the compartments should be measured. The most common location for compartment syndrome is the anterior compartment but all four compartments should be measured. When measuring the compartments, any measurement where the diastolic pressure minus the compartment pressure is < 30 mmHG indicates compartment syndrome. 1,2
References
1. Compartment syndrome. http://www.orthobullets.com. Accessed on 7/22/17.
2. Olsen SA, Glasgow RR. Acute Compartment Syndrome in Lower Extremity Musculoskeletal Trauma. JAAOS 2005; 13: 436-444. -
Question 19 of 44
19. Question
A 46 year-old male presents to the emergency department with severe neck pain after a motor vehicle accident. He was a restrained driver when his car was struck from behind. He denies loss of consciousness and was taken to the ED by ambulance in a hard cervical collar after the accident. On exam his motor and sensation remains intact in his bilateral upper extremities. He denies having any medical conditions or a past history of tobacco use. Sagittal and coronal CT images (figures 1 and 2) show a type II odontoid fracture with 1-2 mm of diastases along the fracture plane. What is the best treatment option for this patient?
Correct
Odontoid (C2) fracture, also known as a dens fracture, accounts for 10-15% of all cervical fractures. The most common mechanism of injury is blunt force trauma to the head causing cervical hyperextension or hyperflexion. Odontoid fractures are often missed in the elderly as fractures can occur after a simple fall. The Anderson-D’Alonzo classification system uses the anatomical location of the fracture line to classify odontoid fractures into three types. Type I is an avulsion fracture off the tip of the odontoid process representing an alar ligament avulsion, type II is a fracture through the waist of the odontoid, and type III is a fracture extending into the vertebral body and C1-C2 articulation. 1,2
Most type I and III fractures can be managed with external immobilization, while treatment for type II fractures remains controversial. The waist of the odontoid has a limited blood supply and less trabecular bone which causes type II fractures to have more trouble healing. Patients prone to nonunion (elderly, smokers, etc) are generally treated with surgery. Patients with no neurological deficits and less than 5 mm of displacement may be treated conservatively with collar immobilization (Philadelphia collar) or a halo vest. A collar has fewer complications and equal union rates compared to a halo vest and therefore is generally the preferable non-operative treatment. Relative indications for surgical treatment include advanced age (over 50 years old), posterior fracture displacement greater than 5 mm, fracture comminution, or history of tobacco use. 1,2References
1. Hsu WK, Anderson P. Odontoid Fractures: Update on Management. JAAOS 2010. Volume 18 (7). 383-394.
2. Odontoid fracture. http://www.orthobullets.com. Accessed on 10/1/2017.Incorrect
Odontoid (C2) fracture, also known as a dens fracture, accounts for 10-15% of all cervical fractures. The most common mechanism of injury is blunt force trauma to the head causing cervical hyperextension or hyperflexion. Odontoid fractures are often missed in the elderly as fractures can occur after a simple fall. The Anderson-D’Alonzo classification system uses the anatomical location of the fracture line to classify odontoid fractures into three types. Type I is an avulsion fracture off the tip of the odontoid process representing an alar ligament avulsion, type II is a fracture through the waist of the odontoid, and type III is a fracture extending into the vertebral body and C1-C2 articulation. 1,2
Most type I and III fractures can be managed with external immobilization, while treatment for type II fractures remains controversial. The waist of the odontoid has a limited blood supply and less trabecular bone which causes type II fractures to have more trouble healing. Patients prone to nonunion (elderly, smokers, etc) are generally treated with surgery. Patients with no neurological deficits and less than 5 mm of displacement may be treated conservatively with collar immobilization (Philadelphia collar) or a halo vest. A collar has fewer complications and equal union rates compared to a halo vest and therefore is generally the preferable non-operative treatment. Relative indications for surgical treatment include advanced age (over 50 years old), posterior fracture displacement greater than 5 mm, fracture comminution, or history of tobacco use. 1,2References
1. Hsu WK, Anderson P. Odontoid Fractures: Update on Management. JAAOS 2010. Volume 18 (7). 383-394.
2. Odontoid fracture. http://www.orthobullets.com. Accessed on 10/1/2017. -
Question 20 of 44
20. Question
A 62 year-old female presents to your office with three months of left shoulder pain. She had a displaced proximal humerus fracture on the left a few years ago that was treated non-operatively and had recovered well until recently. AP x-ray (figure 1) and coronal MRI image (figure 2) of the left shoulder show osteonecrosis of the humeral head with advanced arthritis in the glenohumeral joint. What is the best treatment option?Correct
Traumatic osteonecrosis of the humeral head is a complication that may arise after a proximal humerus fracture. Displaced fractures can disrupt the blood supply to the humeral head resulting in osteonecrosis. Proximal humerus fractures are often described by how many parts or pieces the proximal humerus breaks in. Two, three, or four part fractures occur when displacement and angulation occurs between parts. As the number of fractured parts increases the outcome worsen in terms of the fracture healing successfully. Osteonecrosis of the proximal humerus approaches 100% with a four part fracture dislocation, up to 45% for a displaced four part (without dislocation), and 14% with a displaced three part. 1,2
Symptoms of osteonecrosis of the humeral head include a gradual onset of shoulder pain. The patient may have progressive loss of shoulder motion and strength. Early x-ray findings may be normal. As the disease progresses x-ray may show sclerosis, subchondral fracture (crescent sign), flattening and collapse of the humeral surface, and arthritis. MRI is the study of choice to diagnose osteonecrosis and is usually ordered when the diagnosis is suspected. This particular patient has advanced disease and arthritic changes on both the glenoid and humeral head. The only choice that addresses the patient’s glenohumeral arthritis and osteonecrosis is a total shoulder arthroplasty. 1,2
References
1. Patel S, Colaco HB, Elvey ME, Lee MH. Post-traumatic osteonecrosis of the proximal humerus. Injury. Volume 46, Issue 10, October 2015, Pages 1878-1884
2. Gerber C, Hersche O, Berberat C. The clinical relevance of posttraumatic avascular necrosis of the humeral head. Journal of Shoulder and Elbow Surgery. Volume 7, Issue 6, November–December 1998, Pages 586-590Incorrect
Traumatic osteonecrosis of the humeral head is a complication that may arise after a proximal humerus fracture. Displaced fractures can disrupt the blood supply to the humeral head resulting in osteonecrosis. Proximal humerus fractures are often described by how many parts or pieces the proximal humerus breaks in. Two, three, or four part fractures occur when displacement and angulation occurs between parts. As the number of fractured parts increases the outcome worsen in terms of the fracture healing successfully. Osteonecrosis of the proximal humerus approaches 100% with a four part fracture dislocation, up to 45% for a displaced four part (without dislocation), and 14% with a displaced three part. 1,2
Symptoms of osteonecrosis of the humeral head include a gradual onset of shoulder pain. The patient may have progressive loss of shoulder motion and strength. Early x-ray findings may be normal. As the disease progresses x-ray may show sclerosis, subchondral fracture (crescent sign), flattening and collapse of the humeral surface, and arthritis. MRI is the study of choice to diagnose osteonecrosis and is usually ordered when the diagnosis is suspected. This particular patient has advanced disease and arthritic changes on both the glenoid and humeral head. The only choice that addresses the patient’s glenohumeral arthritis and osteonecrosis is a total shoulder arthroplasty. 1,2
References
1. Patel S, Colaco HB, Elvey ME, Lee MH. Post-traumatic osteonecrosis of the proximal humerus. Injury. Volume 46, Issue 10, October 2015, Pages 1878-1884
2. Gerber C, Hersche O, Berberat C. The clinical relevance of posttraumatic avascular necrosis of the humeral head. Journal of Shoulder and Elbow Surgery. Volume 7, Issue 6, November–December 1998, Pages 586-590 -
Question 21 of 44
21. Question
A 58 year-old female presents to your office with left knee pain for several months. She has a history of a left knee arthroscopic lateral meniscectomy 10 years ago. Bilateral knee AP x-ray (figure 1) taken 5 years ago showed moderate lateral joint line osteoarthritis. Repeat AP x-ray (figure 2) shows that her arthritis has progressed significantly in the lateral compartment of the left knee. Which choice is the biggest predictor of developing long-term arthritis in patients with prior arthroscopic meniscectomy?Correct
Arthroscopic meniscectomy is one of the most common surgical procedures performed in orthopedic practice. The primary function of the medial and lateral meniscus is shock absorption to protect the cartilage of the knee joint. The menisci transmit 50% of the weight bearing load in extension and 85% in flexion. Even small changes to the meniscus, including a partial meniscectomy, can dramatically alter the biomechanics of the knee. 1
There is now a strong push to repair meniscus tears whenever possible as arthroscopic meniscectomy is a known risk factor for the development of knee osteoarthritis. Arthroscopic meniscectomy patients often develop long term pain and functional limitations. In patients that require meniscectomy, the smallest amount of torn meniscus tissue as possible is removed until a stable rim of tissue remains. The amount of tissue removed is directly proportional with the amount of arthritis likely to develop years later. In fact, the amount of meniscal tissue removed is the strongest predictor of long-term onset of arthritis. Other risk factors that have shown a higher rate of knee osteoarthritis after arthroscopic meniscectomy include obesity, female gender, genetic predisposition, simultaneous ACL deficiency, and advanced age and cartilage wear at the time of surgery. 2References
1. http://www.orthobullets.com. Accessed on 1/10/2018
2. Papalia R1, Del Buono A, Osti L, Denaro V, Maffulli N. Meniscectomy as a risk factor for knee osteoarthritis: a systematic review. Br Med Bull. 2011;99:89-106. doi: 10.1093/bmb/ldq043. Epub 2011 Jan 19.Incorrect
Arthroscopic meniscectomy is one of the most common surgical procedures performed in orthopedic practice. The primary function of the medial and lateral meniscus is shock absorption to protect the cartilage of the knee joint. The menisci transmit 50% of the weight bearing load in extension and 85% in flexion. Even small changes to the meniscus, including a partial meniscectomy, can dramatically alter the biomechanics of the knee. 1
There is now a strong push to repair meniscus tears whenever possible as arthroscopic meniscectomy is a known risk factor for the development of knee osteoarthritis. Arthroscopic meniscectomy patients often develop long term pain and functional limitations. In patients that require meniscectomy, the smallest amount of torn meniscus tissue as possible is removed until a stable rim of tissue remains. The amount of tissue removed is directly proportional with the amount of arthritis likely to develop years later. In fact, the amount of meniscal tissue removed is the strongest predictor of long-term onset of arthritis. Other risk factors that have shown a higher rate of knee osteoarthritis after arthroscopic meniscectomy include obesity, female gender, genetic predisposition, simultaneous ACL deficiency, and advanced age and cartilage wear at the time of surgery. 2References
1. http://www.orthobullets.com. Accessed on 1/10/2018
2. Papalia R1, Del Buono A, Osti L, Denaro V, Maffulli N. Meniscectomy as a risk factor for knee osteoarthritis: a systematic review. Br Med Bull. 2011;99:89-106. doi: 10.1093/bmb/ldq043. Epub 2011 Jan 19. -
Question 22 of 44
22. Question
A 32 year-old male presents to the emergency department with severe right foot pain and deformity after an injury 2 hours prior. He was running and may have stepped on an uneven surface when his ankle twisted. He immediately fell to the ground and was unable to bear weight after. He also noticed that his toes pointed inward and the foot was obviously deformed. On exam in the emergency department you note that the foot has obvious medial deviation starting at the midfoot. X-ray and CT reports conclude no fractures or deformities can be seen. However, you note that the 3 dimensional CT reconstructions of the right foot show incongruence of the talonavicular and calcaneocuboid joints (figures above). What is the best treatment option for this rare injury?Correct
The Chopart joint is composed of the talonavicular joint and calcaneocuboid joint (also referred to as the midtarsal joint or the articulation between the midfoot and hindfoot). Chopart was a French surgeon who performed amputations at the midtarsal joint, a procedure still commonly performed in diabetic patients today. A Chopart dislocation occurs when the midfoot (navicular and cuboid) separate from the hindfoot (calcaneus and talus). The Chopart joint is held together by several strong capsular-ligamentous structures including the talonavicular, calcaneonavicular, and calcaneocuboid ligaments. Forceful adduction of the plantar flexed forefoot with a fixed hindfoot can disrupt these ligaments and cause a dislocation with or without a fracture. 1,2
Chopart dislocations can be missed due to the lack of obvious radiographic findings and the fact that it is such a rare injury. Deformity of the foot may be the only obvious sign. Once a Chopart dislocation has been diagnosed urgent closed reduction should be performed in the emergency department. External fixation is then performed in the operating room to maintain the reduction while the ligaments of the midfoot heal. Patients should remain non-weight bearing until the external fixator is removed, typically at 4 weeks postoperatively. The external fixator is then replaced with a short leg cast for an additional 4 weeks. At 8 weeks postoperatively, gradual weight bearing is allowed. 1,2References
1. Benirschke SK, Meinberg E, Anderson SA, Jones CB, Cole PA. Fractures and Dislocations of the Midfoot: Lis Franc and Chopart Injuries. JAAOS. 2012; 94: 1326-37.
2. Schmitt JW, Werner C, Ossendorf C, Wanner GA, Simmen HP. Avulsion Fracture of the Dorsal Talonavicular Ligament: A Subtle Radiographic Sign of Possible Chopart Joint Dislocation. Foot and Ankle International. 2011; 32 (7)Incorrect
The Chopart joint is composed of the talonavicular joint and calcaneocuboid joint (also referred to as the midtarsal joint or the articulation between the midfoot and hindfoot). Chopart was a French surgeon who performed amputations at the midtarsal joint, a procedure still commonly performed in diabetic patients today. A Chopart dislocation occurs when the midfoot (navicular and cuboid) separate from the hindfoot (calcaneus and talus). The Chopart joint is held together by several strong capsular-ligamentous structures including the talonavicular, calcaneonavicular, and calcaneocuboid ligaments. Forceful adduction of the plantar flexed forefoot with a fixed hindfoot can disrupt these ligaments and cause a dislocation with or without a fracture. 1,2
Chopart dislocations can be missed due to the lack of obvious radiographic findings and the fact that it is such a rare injury. Deformity of the foot may be the only obvious sign. Once a Chopart dislocation has been diagnosed urgent closed reduction should be performed in the emergency department. External fixation is then performed in the operating room to maintain the reduction while the ligaments of the midfoot heal. Patients should remain non-weight bearing until the external fixator is removed, typically at 4 weeks postoperatively. The external fixator is then replaced with a short leg cast for an additional 4 weeks. At 8 weeks postoperatively, gradual weight bearing is allowed. 1,2References
1. Benirschke SK, Meinberg E, Anderson SA, Jones CB, Cole PA. Fractures and Dislocations of the Midfoot: Lis Franc and Chopart Injuries. JAAOS. 2012; 94: 1326-37.
2. Schmitt JW, Werner C, Ossendorf C, Wanner GA, Simmen HP. Avulsion Fracture of the Dorsal Talonavicular Ligament: A Subtle Radiographic Sign of Possible Chopart Joint Dislocation. Foot and Ankle International. 2011; 32 (7) -
Question 23 of 44
23. Question
A 27 year-old female presents to the emergency department with increasing lower back and buttock pain for 3 weeks. She admits to frequent intra-venous drug use with a most recent “shoot up” 3 days ago. On exam she has point tenderness over the right sacroiliac (SI) joint. She has no pain with passive range of motion of the right hip. She is febrile with a temperature of 101F and her white blood cell count and inflammatory markers (ESR and CRP) are elevated. Her blood cultures are positive for staphylococcus bacteremia. MRI of the pelvis shows a small right sided SI joint effusion with adjacent soft tissue edema consistent with septic arthritis of the right SI joint (figure 1). What is the next best step in treatment?Correct
Septic arthritis of the SI joint is rare condition representing 1-2% of all forms of septic arthritis and bone infections. The source of infection is thought to be from hematogenous spread. The most common positive physical exam signs include maximal tenderness over the SI joint and increased pain with SI joint compression test. Patients will often have a positive straight leg raise because the sacral nerves running anterior to the SI join may be irritated. MRI is the gold standard test to diagnose an early SI joint infection. Laboratory findings typically include an elevated white blood cell count, elevated inflammatory markers (ESR and CRP), and blood cultures will often be positive for the causative bacterial agent. Identifying the responsible bacteria, if possible, helps guide the most appropriate antibiotic coverage. Obtaining an aspiration is extremely difficult due to the anatomy of the SI joint and is generally not attempted, particularly if blood cultures are positive. Most cases of SI joint infections can be treated successfully with a 4 to 8 week course of antibiotic therapy. Surgical debridement may be performed if pain doesn’t improve despite bed rest and IV antibiotics, if the sepsis worsens, or if a large fluid collection or abscess is present. 1,2
References
1. Hodgson BF. Pyogenic Sacroiliac Joint Infection. Clinical Orthopaedics & Related Research: September 1989. Pages 146-149.
2. Vyskocil JJ, Mcilroy MA, Brennan TA, Wilson FM. Pyogenic Infection of the Sacroiliac Joint. Case Reports and Review of Literature. Medicine 1991. Vol 70. No 3. 188-197.Incorrect
Septic arthritis of the SI joint is rare condition representing 1-2% of all forms of septic arthritis and bone infections. The source of infection is thought to be from hematogenous spread. The most common positive physical exam signs include maximal tenderness over the SI joint and increased pain with SI joint compression test. Patients will often have a positive straight leg raise because the sacral nerves running anterior to the SI join may be irritated. MRI is the gold standard test to diagnose an early SI joint infection. Laboratory findings typically include an elevated white blood cell count, elevated inflammatory markers (ESR and CRP), and blood cultures will often be positive for the causative bacterial agent. Identifying the responsible bacteria, if possible, helps guide the most appropriate antibiotic coverage. Obtaining an aspiration is extremely difficult due to the anatomy of the SI joint and is generally not attempted, particularly if blood cultures are positive. Most cases of SI joint infections can be treated successfully with a 4 to 8 week course of antibiotic therapy. Surgical debridement may be performed if pain doesn’t improve despite bed rest and IV antibiotics, if the sepsis worsens, or if a large fluid collection or abscess is present. 1,2
References
1. Hodgson BF. Pyogenic Sacroiliac Joint Infection. Clinical Orthopaedics & Related Research: September 1989. Pages 146-149.
2. Vyskocil JJ, Mcilroy MA, Brennan TA, Wilson FM. Pyogenic Infection of the Sacroiliac Joint. Case Reports and Review of Literature. Medicine 1991. Vol 70. No 3. 188-197. -
Question 24 of 44
24. Question
A 43 year-old female presents to the office with left shoulder pain after a fall two days ago. She was ice skating and fell with the arm extended. She is now having difficulty lifting the arm overhead. Grashey view x-ray (figure 1) is shown above. What is the best treatment option of this patient?Correct
The patient presents with a minimally displaced greater tuberosity fracture. The patient’s x-rays were read as normal in the emergency room and she was unaware she had a fracture. The greater tuberosity is an attachment site for the supraspinatus and infraspinatus tendons. The greater tuberosity can fracture with a direct blow to the shoulder (an impaction injury) or from a fall on an outstretched hand causing an avulsion injury. A greater tuberosity fracture can also occur in up to 30% of shoulder dislocations. 1,2
A non-displaced fracture may be subtle on x-ray and is often missed. Non-displaced and minimally displaced fractures can be treated in a sling immobilizer to avoid active abduction for 4-6 weeks. Displacement of fractures greater than 5 mm should be fixed with open reduction and internal fixation. Displaced fractures can result in impingement on the under surface of the acromion and a poor functional outcome. Displaced fractures retract posteriorly and superiorly causing impingement in abduction and external rotation. Standard x-rays including grashey AP, outlet view, and axillary view are usually adequate to determine the amount of displacement and indicated treatment. MRI may be utilized for suspected occult fractures not visible on x-ray. 1,2References.
1. Wilcox RB, Arslanian LE, Millett PJ. Management of a patient with an isolated greater tuberosity fracture and rotator cuff tear. JOSPT 2005 35. 521-530.
2. Parsons BO, Klepps SJ, Miller S, Bird J, Gladstone J, Flatow E. Reliability and Reproducibility of Radiographs of Greater Tuberosity Displacement: A Cadaveric Study. JBJS 2005. 87-A (1). 58-65.Incorrect
The patient presents with a minimally displaced greater tuberosity fracture. The patient’s x-rays were read as normal in the emergency room and she was unaware she had a fracture. The greater tuberosity is an attachment site for the supraspinatus and infraspinatus tendons. The greater tuberosity can fracture with a direct blow to the shoulder (an impaction injury) or from a fall on an outstretched hand causing an avulsion injury. A greater tuberosity fracture can also occur in up to 30% of shoulder dislocations. 1,2
A non-displaced fracture may be subtle on x-ray and is often missed. Non-displaced and minimally displaced fractures can be treated in a sling immobilizer to avoid active abduction for 4-6 weeks. Displacement of fractures greater than 5 mm should be fixed with open reduction and internal fixation. Displaced fractures can result in impingement on the under surface of the acromion and a poor functional outcome. Displaced fractures retract posteriorly and superiorly causing impingement in abduction and external rotation. Standard x-rays including grashey AP, outlet view, and axillary view are usually adequate to determine the amount of displacement and indicated treatment. MRI may be utilized for suspected occult fractures not visible on x-ray. 1,2References.
1. Wilcox RB, Arslanian LE, Millett PJ. Management of a patient with an isolated greater tuberosity fracture and rotator cuff tear. JOSPT 2005 35. 521-530.
2. Parsons BO, Klepps SJ, Miller S, Bird J, Gladstone J, Flatow E. Reliability and Reproducibility of Radiographs of Greater Tuberosity Displacement: A Cadaveric Study. JBJS 2005. 87-A (1). 58-65. -
Question 25 of 44
25. Question
A 35 year-old women presents to the office with a several month history of right hip pain. She stopped running recently due to the pain and feeling that the hip was “popping out of place”. On exam she has groin pain with passive hip flexion and internal rotation. She has no lower back pain and no motor weakness in the right lower extremity. AP pelvis and lateral x-ray of the right hip (figures 1 and 2) show a shallow acetabulum, an increased femoral neck-shaft angle, and an abnormal lateral center edge angle consistent with developmental hip dysplasia (DDH). Which choice below is correct regarding lateral center edge angle measurements of the hip?Correct
Developmental hip dysplasia (DDH) is one of the most common causes of hip pain in patients under 50 years of age. Risk factors include female gender, breech position at birth, and a positive family history. DDH is a development disorder in which the hip fails to form in a normal shape. In DDH, the acetabulum is typically shallow, lateralized, and deficient anteriorly and superiorly. On the femoral side the femoral neck is usually in a valgus position (increased neck-shaft angle) and the femoral head is usually small. These anatomical abnormalities cause a decreased contact area between the femoral head and acetabulum which leads to increase motion of the head in the socket. Increased motion leads to early cartilage wear and a higher prevalence of labral tearing. DDH is often missed during infant screening and is most commonly diagnosed when symptoms arise in adulthood. DDH is the most common cause of hip arthritis in women under 50 years of age and up to 10% of all total hips performed in the US are due to DDH. 1,2
AP pelvis and lateral view x-rays are diagnostic of DDH. A common radiographic measurement to diagnose and quantify the severity of DDH is the lateral center edge angle. The angle is formed between a line drawn from the center of the femoral head to the lateral edge of the acetabulum and a vertical line drawn from the center of the femoral head. An angle greater than 25 degrees is considered normal and an angle less than 20 degrees is associated with dysplasia. An angle greater than 39 degrees indicates acetabular over coverage which is seen in pincer type acetabular impingement. 1References
1. Gala L, Clohisy JC, Beaulé PE. Hip Dysplasia in the Young Adult. Journal of Bone & Joint Surgery – American Volume: 6 January 2016 – Volume 98 – Issue 1 – p 63–73
2. Hip Dysplasia. http://hipdysplasia.org. Accessed on 9/30/17.Incorrect
Developmental hip dysplasia (DDH) is one of the most common causes of hip pain in patients under 50 years of age. Risk factors include female gender, breech position at birth, and a positive family history. DDH is a development disorder in which the hip fails to form in a normal shape. In DDH, the acetabulum is typically shallow, lateralized, and deficient anteriorly and superiorly. On the femoral side the femoral neck is usually in a valgus position (increased neck-shaft angle) and the femoral head is usually small. These anatomical abnormalities cause a decreased contact area between the femoral head and acetabulum which leads to increase motion of the head in the socket. Increased motion leads to early cartilage wear and a higher prevalence of labral tearing. DDH is often missed during infant screening and is most commonly diagnosed when symptoms arise in adulthood. DDH is the most common cause of hip arthritis in women under 50 years of age and up to 10% of all total hips performed in the US are due to DDH. 1,2
AP pelvis and lateral view x-rays are diagnostic of DDH. A common radiographic measurement to diagnose and quantify the severity of DDH is the lateral center edge angle. The angle is formed between a line drawn from the center of the femoral head to the lateral edge of the acetabulum and a vertical line drawn from the center of the femoral head. An angle greater than 25 degrees is considered normal and an angle less than 20 degrees is associated with dysplasia. An angle greater than 39 degrees indicates acetabular over coverage which is seen in pincer type acetabular impingement. 1References
1. Gala L, Clohisy JC, Beaulé PE. Hip Dysplasia in the Young Adult. Journal of Bone & Joint Surgery – American Volume: 6 January 2016 – Volume 98 – Issue 1 – p 63–73
2. Hip Dysplasia. http://hipdysplasia.org. Accessed on 9/30/17. -
Question 26 of 44
26. Question
A 67 year-old female presents to the office with activity related right thigh pain for the past 4 months. The pain increases as she gets further into her walks. She now has to stop and sit while walking due to the severe right thigh pain that develops. She denies having a known injury or precipitating event. Her past medical history is significant for hypertension, dyslipidemia, and osteoporosis. Her prescription medications include carvedilol, alendronate, and atorvastatin. On exam she has no pain with hip flexion and rotation or with active hip flexion against resistance. She has no back pain and a negative straight leg raise. AP pelvis and lateral hip x-rays taken in the office are normal. What is the next best step in this patient’s treatment?
Correct
The patient’s history of alendronate (Fosamax) use and progressive thigh pain raises concerns she may have an impending atypical femur fracture. Alendronate is in a class of medications called bisphosphonates that are the most commonly prescribed drugs for the treatment of osteoporosis. A potential complication of long term bisphosphonate use is an atypical femur fracture occurring in the subtrochanteric region of the hip. An atypical fracture is thought to develop from impaired bone remodeling as a result of long term bisphosphonate use. Impaired bone remodeling is thought to cause microdamage to bone and reduced stress-fracture healing which leads to a higher risk of fracture. The incidence of atypical fracture increases over time with continued bisphosphonate use. The incidence with two years of bisphosphonate use is around 2 per 100,000 patients where the incidence after 8 years is around 78 per 100,000. A “drug holiday” is generally recommended in lower risk patients after 5 years of bisphosphonate use due to the concerns of over suppression of bone remodeling. The duration of the holiday is usually correlated with the patient’s fractures risk and based on factors like fall risk, bone mineral density scores, age, and activity level.1,2
If an impending atypical fracture is suspected than x-rays of the proximal femur should be obtained. If x-rays are negative than MRI should be ordered to determine if pre-fracture osseous changes of the proximal femur are present such as a stress reaction or boney edema. If MRI identifies a stress reaction or fracture, the bisphosphonate should be stopped. Incomplete fractures may be treated with limited weight bearing and observation. Although the rate of healing is poor for incomplete atypical fractures with conservative treatment and the trend to treat with surgical stabilization is growing. Open reduction and internal fixation with an intramedullary nail is the most common technique used for surgical stabilization for both complete and incomplete fractures. 1,2
References
1. Unnanuntana A, Saleh A, Mensah KA, Kleimeyer JP, Lane JM. Atypical femur fractures: What do we know about them? JBJS 2013: 95. E8 (1-13)
2. Balach T, Baldwin PC, Intravia J. Atypical femur fractures associated with diphosphonate use. JAAOS 2015; 23: 550-557.Incorrect
The patient’s history of alendronate (Fosamax) use and progressive thigh pain raises concerns she may have an impending atypical femur fracture. Alendronate is in a class of medications called bisphosphonates that are the most commonly prescribed drugs for the treatment of osteoporosis. A potential complication of long term bisphosphonate use is an atypical femur fracture occurring in the subtrochanteric region of the hip. An atypical fracture is thought to develop from impaired bone remodeling as a result of long term bisphosphonate use. Impaired bone remodeling is thought to cause microdamage to bone and reduced stress-fracture healing which leads to a higher risk of fracture. The incidence of atypical fracture increases over time with continued bisphosphonate use. The incidence with two years of bisphosphonate use is around 2 per 100,000 patients where the incidence after 8 years is around 78 per 100,000. A “drug holiday” is generally recommended in lower risk patients after 5 years of bisphosphonate use due to the concerns of over suppression of bone remodeling. The duration of the holiday is usually correlated with the patient’s fractures risk and based on factors like fall risk, bone mineral density scores, age, and activity level.1,2
If an impending atypical fracture is suspected than x-rays of the proximal femur should be obtained. If x-rays are negative than MRI should be ordered to determine if pre-fracture osseous changes of the proximal femur are present such as a stress reaction or boney edema. If MRI identifies a stress reaction or fracture, the bisphosphonate should be stopped. Incomplete fractures may be treated with limited weight bearing and observation. Although the rate of healing is poor for incomplete atypical fractures with conservative treatment and the trend to treat with surgical stabilization is growing. Open reduction and internal fixation with an intramedullary nail is the most common technique used for surgical stabilization for both complete and incomplete fractures. 1,2
References
1. Unnanuntana A, Saleh A, Mensah KA, Kleimeyer JP, Lane JM. Atypical femur fractures: What do we know about them? JBJS 2013: 95. E8 (1-13)
2. Balach T, Baldwin PC, Intravia J. Atypical femur fractures associated with diphosphonate use. JAAOS 2015; 23: 550-557. -
Question 27 of 44
27. Question
A 19-year old female presents to your office 5 days after injuring her right arm during a bike accident. She was thrown over the handle bars and landed on her right elbow. X-rays taken in the ED show a displaced midshaft humerus fracture (figures 1 and 2). She was placed in a coaptation fiberglass splint to the right arm in the ED and was instructed to follow-up with orthopedics. On exam gross motor and sensation in the right arm are intact. Which choice is the best treatment option for this patient?Correct
Most humeral shaft fractures heal well with non-operative treatment. Initial management in the emergency room often includes a coaptation splint or U-shaped splint that goes into the axilla on the medial side and past the deltoid on the lateral side of the arm. The splint is removed during the first follow-up office visit, generally at 4-5 days after the injury. The patient is then converted to a functional brace that provides circumferential compression around the fracture site. Galveston and Sarmiento humeral braces are examples of functional braces as they allow for immediate range of motion of the elbow and wrist. Keeping the arm in adduction is advised as the weight of the arm with gravity helps reduce the shaft fracture. Abduction and forward flexion of the arm creates displacing forces and should be avoided until he fracture heals. Criteria for acceptable alignment in the brace includes less than 20 degrees of anterior angulation, less than 30 degrees of varus/valgus angulation, and less than 3 cm of shortening. 1,2
Radial nerve palsy, a potential complication of humeral shaft fractures, can occur in up to 11% of patients and is more common in comminuted or oblique fracture patterns. Radial nerve palsy spontaneously resolves in 70% of patients in an average of 7 weeks. Absolute indications for surgery include open fracture or neurovascular injury. Relative indications for open reduction and internal fixation include bilateral humerus fractures or polytrauma, pathological fractures, irreducible fractures, and nonunions. Open reduction and internal fixation has a lower risk of complications compared to intrameduallary nails and is the generally preferred surgical option. 1,2References
1. Sarmiento, Augusto; Waddell, James P.; Latta, Loren L. Diaphyseal Humeral Fractures: Treatment Options. Journal of Bone & Joint Surgery – American Volume . 83(10):1566-1579, October 2001.2.
2. Carroll EA, Schweppe M, Langfitt M, Miller AN, Halvorson JJ. Management of Humeral Shaft Fractures. JAAOS 2012; 20: 423-433.Incorrect
Most humeral shaft fractures heal well with non-operative treatment. Initial management in the emergency room often includes a coaptation splint or U-shaped splint that goes into the axilla on the medial side and past the deltoid on the lateral side of the arm. The splint is removed during the first follow-up office visit, generally at 4-5 days after the injury. The patient is then converted to a functional brace that provides circumferential compression around the fracture site. Galveston and Sarmiento humeral braces are examples of functional braces as they allow for immediate range of motion of the elbow and wrist. Keeping the arm in adduction is advised as the weight of the arm with gravity helps reduce the shaft fracture. Abduction and forward flexion of the arm creates displacing forces and should be avoided until he fracture heals. Criteria for acceptable alignment in the brace includes less than 20 degrees of anterior angulation, less than 30 degrees of varus/valgus angulation, and less than 3 cm of shortening. 1,2
Radial nerve palsy, a potential complication of humeral shaft fractures, can occur in up to 11% of patients and is more common in comminuted or oblique fracture patterns. Radial nerve palsy spontaneously resolves in 70% of patients in an average of 7 weeks. Absolute indications for surgery include open fracture or neurovascular injury. Relative indications for open reduction and internal fixation include bilateral humerus fractures or polytrauma, pathological fractures, irreducible fractures, and nonunions. Open reduction and internal fixation has a lower risk of complications compared to intrameduallary nails and is the generally preferred surgical option. 1,2References
1. Sarmiento, Augusto; Waddell, James P.; Latta, Loren L. Diaphyseal Humeral Fractures: Treatment Options. Journal of Bone & Joint Surgery – American Volume . 83(10):1566-1579, October 2001.2.
2. Carroll EA, Schweppe M, Langfitt M, Miller AN, Halvorson JJ. Management of Humeral Shaft Fractures. JAAOS 2012; 20: 423-433. -
Question 28 of 44
28. Question
A 21 year-old female presents with a 2 year history of left wrist pain. She believes the pain started when she injured her wrist lifting weights but she isn’t certain. On exam she has tenderness to palpation over the dorsal wrist and a slight decrease in wrist extension compared to the contralateral side. AP and lateral x-rays (figures 1 and 2) of the left wrist show moderate collapse of the lunate. Coronal MRI image (figure 3) shows avascular necrosis of the lunate with subchondral collapse and fragmentation. These findings are consistent with which diagnosis?Correct
The patient presents with avascular necrosis of the lunate consistent with Kienbock’s disease. Kienbock’s disease can occur as a result of trauma, skeletal variations such as ulnar minus variance, and diseases that disrupt blood supply such as sick cell anemia and lupus. Pain is usually activity related and located over the radiocarpal joint. Early in the disease x-ray findings will be normal and MRI will show reduced vascularity. As the disease progresses the lunate becomes sclerotic, begins to collapse, and degenerative changes occur in the wrist. Early disease can be treated with immobilization and NSAIDs, although further progression is expected and surgery is usually required. Patients with ulnar negative variance may undergo a joint leveling procedure such as a radial shortening osteotomy. Radial shortening osteotomy helps prevent disease progression to degenerative collapse of the proximal row. In advanced disease a more invasive procedure such as a proximal row carpectomy, wrist fusion, or wrist arthroplasty is necessary.
Ollier disease is a condition characterized by multiple benign tumors called enchondromas that develop within bones. Freidberg’s disease is characterized by avascular necrosis of the second metatarsal head. Panner’s Disease is an osteochondrosis of the capitellum and usually seen in pediatric overhead throwing athletes.References
1. Kienbocks disease. http://www.orthobullets.com accessed on 12/16/2017
2. Tadayoshi W, Masatoshi T, Hiroyuki H, Shinichi Y, Noriaki K, Toshihiko O. Long-term Follow-up Of Radial Shortening Osteotomy For Kienböck Disease. The Journal Of Bone & Joint Surgery – Scientific Articles: 01 August 2008 – Volume 90 – Issue 8 – p. 1705-1711.Incorrect
The patient presents with avascular necrosis of the lunate consistent with Kienbock’s disease. Kienbock’s disease can occur as a result of trauma, skeletal variations such as ulnar minus variance, and diseases that disrupt blood supply such as sick cell anemia and lupus. Pain is usually activity related and located over the radiocarpal joint. Early in the disease x-ray findings will be normal and MRI will show reduced vascularity. As the disease progresses the lunate becomes sclerotic, begins to collapse, and degenerative changes occur in the wrist. Early disease can be treated with immobilization and NSAIDs, although further progression is expected and surgery is usually required. Patients with ulnar negative variance may undergo a joint leveling procedure such as a radial shortening osteotomy. Radial shortening osteotomy helps prevent disease progression to degenerative collapse of the proximal row. In advanced disease a more invasive procedure such as a proximal row carpectomy, wrist fusion, or wrist arthroplasty is necessary.
Ollier disease is a condition characterized by multiple benign tumors called enchondromas that develop within bones. Freidberg’s disease is characterized by avascular necrosis of the second metatarsal head. Panner’s Disease is an osteochondrosis of the capitellum and usually seen in pediatric overhead throwing athletes.References
1. Kienbocks disease. http://www.orthobullets.com accessed on 12/16/2017
2. Tadayoshi W, Masatoshi T, Hiroyuki H, Shinichi Y, Noriaki K, Toshihiko O. Long-term Follow-up Of Radial Shortening Osteotomy For Kienböck Disease. The Journal Of Bone & Joint Surgery – Scientific Articles: 01 August 2008 – Volume 90 – Issue 8 – p. 1705-1711. -
Question 29 of 44
29. Question
A 65 year-old female presents to the emergency department with acute on chronic back pain and lower extremity weakness. She also admits to loss of bladder control over the last 8 hours. Sagittal MRI images (figures 1 and 2) show anteriorlisthesis of T11 on T12 resulting in severe canal stenosis and kinking of the spinal cord. There is hyperintense signal in the spinal cord at the thoracic level reflecting spondylotic myelopathy versus myelomacia. Which statement is true regarding this patient’s condition?Correct
The patient’s MRI and clinical presentation are suggestive of Cauda Equina Syndrome (CES). Pain in the back, buttock, and lower extremities, saddle anesthesia, and loss of bowel and bladder function are classic findings. Urinary retention or incontinence are often found in early CES, with retention generally preceding incontinence. Physical exam findings may vary depending on the severity of CES. A careful rectal exam helps make the diagnosis as poor anal sphincter tone is characteristic finding. 1,2
The most devastating consequences of CES can be permanent loss of bowel, bladder, and sexual function. Patients presenting with CES should be treated with an urgent surgical decompression within 48 hours of symptom onset. Patients treated within 48 hours of onset of symptoms were found to have improved sensory and motor recovery as well as urinary and rectal function compared to those treated after 48 hours. There is no difference in neurological recovery between patients treated within 24 hours and those treated within 48 hours. Neurological recovery generally occurs during the 4 month to 13 month post-operative period, although improvements have been seen up to 2-3 years. Urinary incontinence and loss of bowel control are more likely to improve than sexual dysfunction. Perianal numbness is generally the last deficit to recover and the most likely to persist permanently. Chronic back pain, pre-operative rectal dysfunction, and increasing age are poor prognostic indicators for clinical improvement post-operatively. 1,2References
1. Sarvdeep D, Naveen T, Sujit KT,Raj B, Mandeep D. Outcome of spinal decompression in cauda equina syndrome presenting late in developing countries: case series of 50 cases. Eur Spine J. 2011 Dec; 20(12): 2235–2239.
2. Ahn UM, Ahn NU, Buchowski JM, Garrett ES, Sieber AN, Kostuik JP. Cauda equina syndrome secondary to lumbar disc herniation: a meta-analysis of surgical outcomes.Spine. 2000 Jun;25(12):1515-22.Incorrect
The patient’s MRI and clinical presentation are suggestive of Cauda Equina Syndrome (CES). Pain in the back, buttock, and lower extremities, saddle anesthesia, and loss of bowel and bladder function are classic findings. Urinary retention or incontinence are often found in early CES, with retention generally preceding incontinence. Physical exam findings may vary depending on the severity of CES. A careful rectal exam helps make the diagnosis as poor anal sphincter tone is characteristic finding. 1,2
The most devastating consequences of CES can be permanent loss of bowel, bladder, and sexual function. Patients presenting with CES should be treated with an urgent surgical decompression within 48 hours of symptom onset. Patients treated within 48 hours of onset of symptoms were found to have improved sensory and motor recovery as well as urinary and rectal function compared to those treated after 48 hours. There is no difference in neurological recovery between patients treated within 24 hours and those treated within 48 hours. Neurological recovery generally occurs during the 4 month to 13 month post-operative period, although improvements have been seen up to 2-3 years. Urinary incontinence and loss of bowel control are more likely to improve than sexual dysfunction. Perianal numbness is generally the last deficit to recover and the most likely to persist permanently. Chronic back pain, pre-operative rectal dysfunction, and increasing age are poor prognostic indicators for clinical improvement post-operatively. 1,2References
1. Sarvdeep D, Naveen T, Sujit KT,Raj B, Mandeep D. Outcome of spinal decompression in cauda equina syndrome presenting late in developing countries: case series of 50 cases. Eur Spine J. 2011 Dec; 20(12): 2235–2239.
2. Ahn UM, Ahn NU, Buchowski JM, Garrett ES, Sieber AN, Kostuik JP. Cauda equina syndrome secondary to lumbar disc herniation: a meta-analysis of surgical outcomes.Spine. 2000 Jun;25(12):1515-22. -
Question 30 of 44
30. Question
A 58 year-old female presents to the office with 3 months of left wrist pain. She slipped on ice and fell on her wrist 3 months ago but did not seek treatment at that time. Initially the wrist was a little sore but the pain never really went away. AP, lateral, and scaphoid view x-rays (figures 1, 2, and 3) taken in the office show a scaphoid fracture with minimal healing noted. The scapholunate angle appears normal. CT shows a comminuted scaphoid fracture that’s extends from the waist and travels proximally. What is the best treatment option for this patient?Correct
The scaphoid is the most common carpal bone to fracture. Nondisplaced scaphoid fractures can be missed when initial x-rays are read as normal and/or the injury is believed to be a wrist sprain. In fact up to 30% of scaphoid fractures may not be detectable on initial x-rays. Because the scaphoid has a poor blood supply to the waist and proximal pole, failure to treat fractures early can increase the likelihood of nonunion and osteonecrosis. This is why an MRI is often ordered when patients present acutely with anatomical snuffbox pain and negative x-rays, or an apparent occult scaphoid fracture. When patients present with a remote history of a fall and apparent scaphoid nonunion on x-ray, CT is the study of choice. CT offers the best study to visualize if bony healing across the fracture has occurred. Evidence or absence of healing is crucial in guiding further treatment. 1,2
If patients are still symptomatic and there is no evidence of fracture healing at 3 months then surgical treatment should be discussed. Left untreated scaphoid nonunions can go on to osteonecrosis, carpal instability, carpal collapse, and late wrist arthritis. Scaphoid nonunions involving the proximal pole should be fixed surgically with a vascularized bone graft and screw fixation. Bone grafting has been shown to increase the rate of union over screw fixation alone. More invasive procedures such as proximal row carpectomy, intercapral fusion, scaphoid excision, and wrist fusion are reserved as salvage procedures when a chronic scaphoid nonunion has advanced to wrist arthritis. 2References
1. Merrel GA, Wolf SW, Slade JF. Treatment of Scaphoid Nonunions: Quantitative Meta-analysis of the Literature. The Journal of Hand Surgery 2002; 27A. 685-691.
2. Raju PK, Kini SG. Fixation techniques for nonunion of the scaphoid. Journal of Orthopedic Surgery 2011; 19(1): 80-4.Incorrect
The scaphoid is the most common carpal bone to fracture. Nondisplaced scaphoid fractures can be missed when initial x-rays are read as normal and/or the injury is believed to be a wrist sprain. In fact up to 30% of scaphoid fractures may not be detectable on initial x-rays. Because the scaphoid has a poor blood supply to the waist and proximal pole, failure to treat fractures early can increase the likelihood of nonunion and osteonecrosis. This is why an MRI is often ordered when patients present acutely with anatomical snuffbox pain and negative x-rays, or an apparent occult scaphoid fracture. When patients present with a remote history of a fall and apparent scaphoid nonunion on x-ray, CT is the study of choice. CT offers the best study to visualize if bony healing across the fracture has occurred. Evidence or absence of healing is crucial in guiding further treatment. 1,2
If patients are still symptomatic and there is no evidence of fracture healing at 3 months then surgical treatment should be discussed. Left untreated scaphoid nonunions can go on to osteonecrosis, carpal instability, carpal collapse, and late wrist arthritis. Scaphoid nonunions involving the proximal pole should be fixed surgically with a vascularized bone graft and screw fixation. Bone grafting has been shown to increase the rate of union over screw fixation alone. More invasive procedures such as proximal row carpectomy, intercapral fusion, scaphoid excision, and wrist fusion are reserved as salvage procedures when a chronic scaphoid nonunion has advanced to wrist arthritis. 2References
1. Merrel GA, Wolf SW, Slade JF. Treatment of Scaphoid Nonunions: Quantitative Meta-analysis of the Literature. The Journal of Hand Surgery 2002; 27A. 685-691.
2. Raju PK, Kini SG. Fixation techniques for nonunion of the scaphoid. Journal of Orthopedic Surgery 2011; 19(1): 80-4. -
Question 31 of 44
31. Question
Patient A is a 70 year-old male who has been followed for the last few years with left shoulder pain. AP x-ray of the left shoulder shows severe shoulder OA. He has had short term relief with intra-articular steroid injections, rest, and NSAIDS. On exam he shows good range of motion of the shoulder both passively and actively. MRI done one month ago reveals an intact rotator cuff. Patient B is a 74 year-old male with several years of right shoulder pain and weakness. He has a history of a chronically torn rotator cuff that was debrided a few years ago arthroscopically. On exam he has near full passive range of motion but is very weak with isolated supraspinatus muscle testing. He has failed the same conservative treatments as Patient A. Which choice below offers the best treatment option for each patient?
Correct
Arthritis of shoulder, or glenohumeral joint, can cause significant pain, loss of motion, and disability. There are several causes of shoulder arthritis including primary and secondary osteoarthritis, rotator cuff arthropathy, rheumatoid arthritis, avascular necrosis, and connective tissue disorders. Primary arthritis is an age related cartilage degeneration over time and secondary arthritis is caused by a prior injury that results in a degenerative cascade. Shoulder arthritis and rotator cuff pathology are prevalent in over 90% of patients with rheumatoid arthritis. Common symptoms include night pain, decreased range of motion, particularly external rotation, and painful range of motion. 1,2
The first line treatments in all patients who present with shoulder arthritis include NSAID’s, DMARD’s for rheumatoid arthritis, activity modification, physical therapy and intra-articular injections. The three main surgical options available once conservative treatments have failed include hemiarthroplasty, total shoulder arthroplasty, and reverse total shoulder arthroplasty. Important factors considered when choosing which treatment option is best includes the patient’s age and activity level, degree of arthritis on the humeral surface and glenoid, and the integrity of the rotator cuff on MRI. Younger patients, such as laborers or athletes, place a higher demand on the shoulder and are more prone to complications with total shoulder arthroplasty. Younger patients are also more likely to have chondral wear on the humeral surface and not on the glenoid, which makes a hemiarthroplasty (replace or resurface the humeral side only) a good option for this patient population. Patients with arthritis on both the humeral and glenoid surfaces who have an intact or repairable rotator cuff are good candidates for a total shoulder arthroplasty. Less than 10% of patients who present with shoulder arthritis have a rotator cuff tear so a total shoulder arthroplasty is the most common procedure performed of these three surgical options. In patients with a total shoulder, the rotator cuff keeps the humerus depressed in the glenoid and allows for active shoulder motion. Patients with a deficient rotator cuff will likely fail a total shoulder replacement as the humerus tends to migrate superiorly and forward elevation of the shoulder is lost (pseudoparesis). Therefore patients with a deficient rotator cuff, or rotator cuff arthropathy, are considered for a reverse total shoulder procedure. A reverse total shoulder replacement relies on the deltoid to move the shoulder as the deficient rotator cuff no longer can. Reverse total shoulder replacement is indicated in patients over 70 years of age with severe rotator cuff arthropathy and in patients who fail total shoulder arthroplasty. 1,2References
1. Wiater JM, Fabing MH. Shoulder Arthroplasty: Prosthetic Options and Indications. JAAOS 2009; 17: 415-425.
2. Denard PJ, Wirth MA, Orfaly RM. Current Concept Review: Management of Glenohumeral Arthritis in the Young Adult. JBJS 2011; 93: 885-92.Incorrect
Arthritis of shoulder, or glenohumeral joint, can cause significant pain, loss of motion, and disability. There are several causes of shoulder arthritis including primary and secondary osteoarthritis, rotator cuff arthropathy, rheumatoid arthritis, avascular necrosis, and connective tissue disorders. Primary arthritis is an age related cartilage degeneration over time and secondary arthritis is caused by a prior injury that results in a degenerative cascade. Shoulder arthritis and rotator cuff pathology are prevalent in over 90% of patients with rheumatoid arthritis. Common symptoms include night pain, decreased range of motion, particularly external rotation, and painful range of motion. 1,2
The first line treatments in all patients who present with shoulder arthritis include NSAID’s, DMARD’s for rheumatoid arthritis, activity modification, physical therapy and intra-articular injections. The three main surgical options available once conservative treatments have failed include hemiarthroplasty, total shoulder arthroplasty, and reverse total shoulder arthroplasty. Important factors considered when choosing which treatment option is best includes the patient’s age and activity level, degree of arthritis on the humeral surface and glenoid, and the integrity of the rotator cuff on MRI. Younger patients, such as laborers or athletes, place a higher demand on the shoulder and are more prone to complications with total shoulder arthroplasty. Younger patients are also more likely to have chondral wear on the humeral surface and not on the glenoid, which makes a hemiarthroplasty (replace or resurface the humeral side only) a good option for this patient population. Patients with arthritis on both the humeral and glenoid surfaces who have an intact or repairable rotator cuff are good candidates for a total shoulder arthroplasty. Less than 10% of patients who present with shoulder arthritis have a rotator cuff tear so a total shoulder arthroplasty is the most common procedure performed of these three surgical options. In patients with a total shoulder, the rotator cuff keeps the humerus depressed in the glenoid and allows for active shoulder motion. Patients with a deficient rotator cuff will likely fail a total shoulder replacement as the humerus tends to migrate superiorly and forward elevation of the shoulder is lost (pseudoparesis). Therefore patients with a deficient rotator cuff, or rotator cuff arthropathy, are considered for a reverse total shoulder procedure. A reverse total shoulder replacement relies on the deltoid to move the shoulder as the deficient rotator cuff no longer can. Reverse total shoulder replacement is indicated in patients over 70 years of age with severe rotator cuff arthropathy and in patients who fail total shoulder arthroplasty. 1,2References
1. Wiater JM, Fabing MH. Shoulder Arthroplasty: Prosthetic Options and Indications. JAAOS 2009; 17: 415-425.
2. Denard PJ, Wirth MA, Orfaly RM. Current Concept Review: Management of Glenohumeral Arthritis in the Young Adult. JBJS 2011; 93: 885-92. -
Question 32 of 44
32. Question
A 61 year-old male presents to the office with chronic left foot pain. He has pain over the medial ankle, especially when walking on uneven ground. Physical exam reveals a rigid flat foot deformity with tenderness along the posterior tibial tendon and pain with subtalar motion. He notes to have hindfoot valgus and forefoot pronation that cannot be corrected passively beyond neutral. AP and lateral x-rays of the left foot (figures 1 and 2) show an increased talo-first metatarsal angle (Meary angle) and loss of arch height. He has failed at least 3 years of conservative treatment including NSAIDS, rest, steroid injections, orthotics, and bracing. What surgical option would most commonly be recommended for this patient?Correct
The most common cause of an adult acquired flat foot deformity (AAFFD) is posterior tibial tendon dysfunction. As AAFFD progress the midfoot and forefoot abduct and the hindfoot moves into a valgus position. The deltoid ligament becomes incompetent late in the condition as the hindfoot moves further into a valgus position. A tight Achilles tendon can develop and worsen the deformity. Treatment largely depends on whether the flat foot deformity is rigid or flexible. A rigid flatfoot deformity cannot be corrected passively beyond neutral which is a distinguishable feature from a flexible deformity. On exam of a rigid deformity forefoot pronation cannot be corrected passively with the heel in valgus. The rigidity of the tanlonavicular, subtalar, and calcaneocuboid joints leads to fixed hindfoot valgus and midfoot abduction.
Patients with posterior tibial tendon dysfunction, a flat foot deformity, and a flexible hindfoot are candidates for tendon transfers including posterior tibial tendon debridement and flexor digitorum longus (FDL) tendon transfer or FDL tendon transfer to the navicular, spring ligament repair, calcaneal osteotomy, and Achilles lengthening. Patients with rigid hindfoot valgus and forefoot abduction deformities require a fusion procedure, most commonly a triple arthrodesis. A triple arthrodesis involves fusion of the talonavicular, subtalar, and calcaneocuboid joints. The goal of fusion is to create a neutral hindfoot and forefoot.References
1. Vulcano E, Deland JT, Ellis SJ. Approach and treatment of the adult acquired flatfoot deformity. Curr Rev Musculoskelet Med. 2013 Dec; 6(4): 294–303.
2. Abousayed M, Alley M, Shakked R, Rosenbaum A. Adult-Acquired Flatfoot Deformity: Etiology, Diagnosis, and Management. JBJS Reviews: August 2017 – Volume 5 – Issue 8 – pIncorrect
The most common cause of an adult acquired flat foot deformity (AAFFD) is posterior tibial tendon dysfunction. As AAFFD progress the midfoot and forefoot abduct and the hindfoot moves into a valgus position. The deltoid ligament becomes incompetent late in the condition as the hindfoot moves further into a valgus position. A tight Achilles tendon can develop and worsen the deformity. Treatment largely depends on whether the flat foot deformity is rigid or flexible. A rigid flatfoot deformity cannot be corrected passively beyond neutral which is a distinguishable feature from a flexible deformity. On exam of a rigid deformity forefoot pronation cannot be corrected passively with the heel in valgus. The rigidity of the tanlonavicular, subtalar, and calcaneocuboid joints leads to fixed hindfoot valgus and midfoot abduction.
Patients with posterior tibial tendon dysfunction, a flat foot deformity, and a flexible hindfoot are candidates for tendon transfers including posterior tibial tendon debridement and flexor digitorum longus (FDL) tendon transfer or FDL tendon transfer to the navicular, spring ligament repair, calcaneal osteotomy, and Achilles lengthening. Patients with rigid hindfoot valgus and forefoot abduction deformities require a fusion procedure, most commonly a triple arthrodesis. A triple arthrodesis involves fusion of the talonavicular, subtalar, and calcaneocuboid joints. The goal of fusion is to create a neutral hindfoot and forefoot.References
1. Vulcano E, Deland JT, Ellis SJ. Approach and treatment of the adult acquired flatfoot deformity. Curr Rev Musculoskelet Med. 2013 Dec; 6(4): 294–303.
2. Abousayed M, Alley M, Shakked R, Rosenbaum A. Adult-Acquired Flatfoot Deformity: Etiology, Diagnosis, and Management. JBJS Reviews: August 2017 – Volume 5 – Issue 8 – p -
Question 33 of 44
33. Question
A 65 year-old male presents with left thigh pain 2 years after a total hip replacement. He is a postal worker and has noticed the thigh pain over the last few months when he is walking with a heavy bag. He denies pain at rest or having fevers, chills, or sweats. He was doing great after the surgery and admits this is the first time he has had pain. On exam he has no pain with both passive and active range of motion of the hip. He has no pain with hip flexion against resistance or tenderness to palpation over the greater trochanter. The patient’s presentation is suspicious for which diagnosis?
Correct
Identifying the cause of new onset hip pain in patients who have had a total hip replacement can be very difficult. The majority of total hip replacement patients (90%) have excellent pain control and improved activity level. Pain that never improves after surgery is likely a complication of the surgery itself such as infection, instability, fracture, or poor implant alignment. Persistent pain can also be a sign of other pathology that may have been missed such has lumbar or sacroiliac joint pathology. Pain at rest that increases while walking is more common with loosening of the prosthetic components. Activity related pain and pain with resisted hip flexion are signs of tendinitis or bursitis of the hip. Pain to palpation over the greater trochanter most likely indicates trochanteric bursitis. Pain at night or at rest is common with infection and tumors. Groin pain is more indicative of problems with the acetabular component where thigh pain is more indicative of problems with femoral stem. 1,2
Aseptic loosening is the most common cause of pain after total hip and should be suspected in this patient. X-rays may be normal or show radiolucent lines and/or shifting of the prosthetic components with aseptic loosening. The first step in the work-up of hip pain is generally lab testing including ordering ESR and CRP (inflammatory markers) to rule out infection. If the inflammatory markers are elevated and the suspicion of hip infection is high, a joint aspiration should then be performed. A bone scan can be ordered when infection is ruled out. Bone scan can pick up areas of increase bone activity that may suggest prosthetic loosening. Although results should be interpreted carefully as an increase uptake on bone scan can be seen up to two years after total hip replacement in patients with no hip pain. 1,2References
1. Duffy, PJ, Masri BA, Garbuz DS. Evaluation of Patients with Pain Following Total Hip Replacement. JBJS 2005. 87(11):2566-2575.
2. Robbins GM, Masri BA, Garbuz DS, Duncan CP. Evaluation of Pain in Patients with Apparently Solidly Fixed Total Hip Arthroplasty Components. JAAOS 2002; 10: 866-94.Incorrect
Identifying the cause of new onset hip pain in patients who have had a total hip replacement can be very difficult. The majority of total hip replacement patients (90%) have excellent pain control and improved activity level. Pain that never improves after surgery is likely a complication of the surgery itself such as infection, instability, fracture, or poor implant alignment. Persistent pain can also be a sign of other pathology that may have been missed such has lumbar or sacroiliac joint pathology. Pain at rest that increases while walking is more common with loosening of the prosthetic components. Activity related pain and pain with resisted hip flexion are signs of tendinitis or bursitis of the hip. Pain to palpation over the greater trochanter most likely indicates trochanteric bursitis. Pain at night or at rest is common with infection and tumors. Groin pain is more indicative of problems with the acetabular component where thigh pain is more indicative of problems with femoral stem. 1,2
Aseptic loosening is the most common cause of pain after total hip and should be suspected in this patient. X-rays may be normal or show radiolucent lines and/or shifting of the prosthetic components with aseptic loosening. The first step in the work-up of hip pain is generally lab testing including ordering ESR and CRP (inflammatory markers) to rule out infection. If the inflammatory markers are elevated and the suspicion of hip infection is high, a joint aspiration should then be performed. A bone scan can be ordered when infection is ruled out. Bone scan can pick up areas of increase bone activity that may suggest prosthetic loosening. Although results should be interpreted carefully as an increase uptake on bone scan can be seen up to two years after total hip replacement in patients with no hip pain. 1,2References
1. Duffy, PJ, Masri BA, Garbuz DS. Evaluation of Patients with Pain Following Total Hip Replacement. JBJS 2005. 87(11):2566-2575.
2. Robbins GM, Masri BA, Garbuz DS, Duncan CP. Evaluation of Pain in Patients with Apparently Solidly Fixed Total Hip Arthroplasty Components. JAAOS 2002; 10: 866-94. -
Question 34 of 44
34. Question
An 11 year-old female presents to your office with right elbow pain after a cheerleading injury 2 days ago. She fell from 4 feet in the air and landed directly on her outstretched arm. She is now having pain and difficultly moving the elbow. She has a cheerleading competition in two days and is hoping for clearance to get back to practice. On exam she has tenderness to palpation of the posterior and medial elbow with mild generalized elbow swelling. Bilateral AP elbow x-rays are performed for comparison (figure 1). The injured right elbow appears to have some displacement of the medial apophysis. What is the next best step in this patient’s treatment?Correct
The medial epicondyle apophysis is on the posterior medial elbow and is an attachment site for the wrist flexors. A fall on an outstretched arm causes the flexor-pronator mass and MCL to pull on the apophysis which can result in fracture. With enough force, the medial apophysis can avulse off the distal humerus. Fracture displacement can be very subtle and bilateral elbow x-rays are commonly performed to compare the contralateral apophysis. The peek age incidence of medial apophyseal fractures is ages 11 to 12 years old, which is a period of increased skeletal growth. The medial apophysis fuses at age 15 on average. 1,2
The bilateral comparison AP x-rays in this patient show minimal medial apophyseal displacement of 2-3 mm. Displacement of <5 mm should be treated non-operatively. A brief period (generally 2-3 weeks) of immobilization is recommended for these injuries. A long arm cast is a good option for potentially non-compliant children, as in this case of a young girl who is anxious to start cheerleading again. Elbow stiffness is a common complication associated with elbow fractures so gentle range of motion should be initiated early when the cast comes off in 2-3 weeks. A sling can be used when the cast comes off to facilitate daily range of motion exercises. Patients should be warned that these fractures can cause some loss of elbow extension and valgus instability, although most will never notice these complications functionally. 1References
1. Beaty JH, and Kasser JR. The Elbow: Physeal Fractures, Apophyseal Injuries of the Distal Humerus, Osteonecrosis of the Trochlea, and T-Condylar Fractures. Rockwood and Wilken’s: Fractures in Children, 6th Edition. Pages 614-620. Lippincott Williams and Wilkins. Philadelphia, PA. 2006.
2. Medial Epicondylar Factures: http://www.orthobullets.com. Accessed on 2/1Incorrect
The medial epicondyle apophysis is on the posterior medial elbow and is an attachment site for the wrist flexors. A fall on an outstretched arm causes the flexor-pronator mass and MCL to pull on the apophysis which can result in fracture. With enough force, the medial apophysis can avulse off the distal humerus. Fracture displacement can be very subtle and bilateral elbow x-rays are commonly performed to compare the contralateral apophysis. The peek age incidence of medial apophyseal fractures is ages 11 to 12 years old, which is a period of increased skeletal growth. The medial apophysis fuses at age 15 on average. 1,2
The bilateral comparison AP x-rays in this patient show minimal medial apophyseal displacement of 2-3 mm. Displacement of <5 mm should be treated non-operatively. A brief period (generally 2-3 weeks) of immobilization is recommended for these injuries. A long arm cast is a good option for potentially non-compliant children, as in this case of a young girl who is anxious to start cheerleading again. Elbow stiffness is a common complication associated with elbow fractures so gentle range of motion should be initiated early when the cast comes off in 2-3 weeks. A sling can be used when the cast comes off to facilitate daily range of motion exercises. Patients should be warned that these fractures can cause some loss of elbow extension and valgus instability, although most will never notice these complications functionally. 1References
1. Beaty JH, and Kasser JR. The Elbow: Physeal Fractures, Apophyseal Injuries of the Distal Humerus, Osteonecrosis of the Trochlea, and T-Condylar Fractures. Rockwood and Wilken’s: Fractures in Children, 6th Edition. Pages 614-620. Lippincott Williams and Wilkins. Philadelphia, PA. 2006.
2. Medial Epicondylar Factures: http://www.orthobullets.com. Accessed on 2/1 -
Question 35 of 44
35. Question
A 19 year-old female presents to the office with chronic instability of the left ankle. She is an avid soccer player and has had multiple sprains over the last few years. She has tried bracing and ankle taping before games and still has instability episodes. She has also had several months of physical therapy without resolution of symptoms. On exam the patient has an increased anterior drawer test and slight talar tilt compared to the contralateral ankle. X-rays of the left ankle show no abnormalities. She was seen by a foot and ankle specialist recently who recommended a surgical procedure called a Modified Brostrum procedure to help stabilize her ankle for a return to sports. Which ligaments does the modified Brostrum procedure repair in order to correct ankle instability?
Correct
Lateral ankle sprains are one of the most common injuries seen in orthopedic practice. In fact, an estimated 9 million ankle sprains occur annually in the US. Most mild ankle sprains heal uneventfully in 4-6 weeks with a brief period of immobilization. Severe ankle sprains may take up to 3 months to heal and are more prone to recurrent instability, particularly if not rehabbed appropriately. A small percentage of patients who sustain an ankle sprain will go on to have chronic instability with feelings of the ankle “giving way”. Patients will often complain more of instability than pain with chronic symptoms. When patients have chronic symptoms, associated injuries such as osteochondral injuries, arthritis, loose bodies, and peroneal tendinopathy must be ruled out. MRI is often ordered to rule out other pathology. 1,2
The most common surgical procedure performed for chronic instability is called a modified Brostrum procedure. The modified Brostrum procedure involves repairing or tightening the anterior talofibular ligament (ATFL) and the calcaneofibular ligament (CFL) to the fibula. Isolated injury to the ATFL occurs with most (65%) of ankle sprains. Both the ATFL and the CFL are involved in 20% of ankle sprains. The modified Brostrum procedure tightens the lateral ligaments which reduces lateral instability when the foot goes in plantar flexion and inversion (typical mechanism of an ankle sprain). Postoperatively, a 2-4 week period of immobilization followed by 2-3 months of rehabilitation is recommended. Athletes often return to sports 4 months after the procedure. 2References
1. Polzer H, Kanz KG, Prall WC, Haasters F, Ockert B, Mutschler W, Grote S. Diagnosis and treatment of acute ankle injuries: development of an evidence-based algorithm. Orthop Rev (Pavia). 2012 Jan 2; 4(1).
2. Wukich DK, Tuason DA. Diagnosis and Treatment of Chronic Ankle Pain. JBJS 2010; 92: 2002-2016.Incorrect
Lateral ankle sprains are one of the most common injuries seen in orthopedic practice. In fact, an estimated 9 million ankle sprains occur annually in the US. Most mild ankle sprains heal uneventfully in 4-6 weeks with a brief period of immobilization. Severe ankle sprains may take up to 3 months to heal and are more prone to recurrent instability, particularly if not rehabbed appropriately. A small percentage of patients who sustain an ankle sprain will go on to have chronic instability with feelings of the ankle “giving way”. Patients will often complain more of instability than pain with chronic symptoms. When patients have chronic symptoms, associated injuries such as osteochondral injuries, arthritis, loose bodies, and peroneal tendinopathy must be ruled out. MRI is often ordered to rule out other pathology. 1,2
The most common surgical procedure performed for chronic instability is called a modified Brostrum procedure. The modified Brostrum procedure involves repairing or tightening the anterior talofibular ligament (ATFL) and the calcaneofibular ligament (CFL) to the fibula. Isolated injury to the ATFL occurs with most (65%) of ankle sprains. Both the ATFL and the CFL are involved in 20% of ankle sprains. The modified Brostrum procedure tightens the lateral ligaments which reduces lateral instability when the foot goes in plantar flexion and inversion (typical mechanism of an ankle sprain). Postoperatively, a 2-4 week period of immobilization followed by 2-3 months of rehabilitation is recommended. Athletes often return to sports 4 months after the procedure. 2References
1. Polzer H, Kanz KG, Prall WC, Haasters F, Ockert B, Mutschler W, Grote S. Diagnosis and treatment of acute ankle injuries: development of an evidence-based algorithm. Orthop Rev (Pavia). 2012 Jan 2; 4(1).
2. Wukich DK, Tuason DA. Diagnosis and Treatment of Chronic Ankle Pain. JBJS 2010; 92: 2002-2016. -
Question 36 of 44
36. Question
A 48 year-old female presents to your office with severe right shoulder and neck pain for 2-3 months. She denies any injury or known precipitating event. The pain seems to be worse with shoulder and neck motion. Most of the pain is located in the lateral arm and neck area. The arm pain seems to stop at the mid arm and she denies numbness or tingling in the hands. She is difficult to examine secondary to pain, though she seems to have full passive and active motion of the shoulder and neck. She has mildly positive shoulder impingement signs and a negative Spurling’s to the right arm. Which physical exam finding would most likely indicate the pain is coming from her neck?
Correct
A herniated cervical disc can compress the nerves roots exiting the neck causing radiating pain to the shoulder and arm, or cervical radiculopathy. The pattern of radiculopathy doesn’t always follow a classic dermatomal pattern and may be confused with pain originating from the shoulder. Confusing things further is the fact that some patients presenting with cervical radiculopathy can also have painful shoulder impingement. A careful review of the patient’s symptoms and physical exam findings is crucial to distinguish between cervical spine and shoulder pathologies. Electromyography and advanced imaging of the shoulder and cervical spine with MRI can ultimately determine the source of pain but are too costly to be used as first line screening tests.
Turning the patient’s neck to the affected side can decrease the space in the neural foramen, which increases nerve root compression and pain. A positive Spurling’s maneuver, performed by gently moving the patient’s neck in extension, leaning the head to the affected side, and placing slight axial compression, is very specific for cervical radiculopathy. However, Spurling’s has a relatively low sensitivity for cervical radiculopathy with one study showing a positive test found in only 30% of patients with positive electromyography (EMG) studies. Abducting the affected arm offers relief of the radicular symptoms by releasing tension off the compressed nerve root, also called the shoulder abduction relief sign. An arm squeeze test involves placing a moderate squeeze to the mid biceps and triceps area on the affected side. The rationale behind the test is that the musculocutaneous, radial, ulnar, and the median nerves are relatively superficial in that area and an arm squeeze stretches the nerve which worsens the compression of the nerve roots from C5-T1. Cervical traction should help relieve radicular symptoms from the neck by opening up the neuroforamen and reducing compression on the nerve root. 1,2References
1. Tong HC1, Haig AJ, Yamakawa K. The Spurling test and cervical radiculopathy. Spine (Phila Pa 1976). 2002 Jan 15;27(2):156-9.2. Gumina S, Carbone S, Albino P, Gurzi M, Postacchini F. Arm Squeeze Test: a new clinical test to distinguish neck from shoulder pain. Eur Spine J. 2013 Jul; 22(7): 1558–1563.
Incorrect
A herniated cervical disc can compress the nerves roots exiting the neck causing radiating pain to the shoulder and arm, or cervical radiculopathy. The pattern of radiculopathy doesn’t always follow a classic dermatomal pattern and may be confused with pain originating from the shoulder. Confusing things further is the fact that some patients presenting with cervical radiculopathy can also have painful shoulder impingement. A careful review of the patient’s symptoms and physical exam findings is crucial to distinguish between cervical spine and shoulder pathologies. Electromyography and advanced imaging of the shoulder and cervical spine with MRI can ultimately determine the source of pain but are too costly to be used as first line screening tests.
Turning the patient’s neck to the affected side can decrease the space in the neural foramen, which increases nerve root compression and pain. A positive Spurling’s maneuver, performed by gently moving the patient’s neck in extension, leaning the head to the affected side, and placing slight axial compression, is very specific for cervical radiculopathy. However, Spurling’s has a relatively low sensitivity for cervical radiculopathy with one study showing a positive test found in only 30% of patients with positive electromyography (EMG) studies. Abducting the affected arm offers relief of the radicular symptoms by releasing tension off the compressed nerve root, also called the shoulder abduction relief sign. An arm squeeze test involves placing a moderate squeeze to the mid biceps and triceps area on the affected side. The rationale behind the test is that the musculocutaneous, radial, ulnar, and the median nerves are relatively superficial in that area and an arm squeeze stretches the nerve which worsens the compression of the nerve roots from C5-T1. Cervical traction should help relieve radicular symptoms from the neck by opening up the neuroforamen and reducing compression on the nerve root. 1,2References
1. Tong HC1, Haig AJ, Yamakawa K. The Spurling test and cervical radiculopathy. Spine (Phila Pa 1976). 2002 Jan 15;27(2):156-9.2. Gumina S, Carbone S, Albino P, Gurzi M, Postacchini F. Arm Squeeze Test: a new clinical test to distinguish neck from shoulder pain. Eur Spine J. 2013 Jul; 22(7): 1558–1563.
-
Question 37 of 44
37. Question
A 31 year-old female presents to the office with right foot pain after a fall. She tripped while walking down the stairs 2 days ago and her ankle twisted from under her. She has had difficulty bearing weight after the injury and has developed bruising over the lateral foot. AP, oblique, and lateral x-rays of the right foot show a tuberosity avulsion fracture of the proximal 5th metatarsal. Which structure attaches to the tuberosity causing an avulsion force to the fracture?Correct
The majority of 5th metatarsal fractures occur proximally at the tuberosity. Proximal avulsion fractures are often called “pseudo Jones” fractures where a true jones fracture occurs at the metaphyseal-diaphyseal junction and area of poor circulation. Proximal fractures result from a forceful inversion of the foot in a plantar flexed position, or the same mechanism as a common lateral ankle sprain. For this reason, anyone who presents with a lateral ankle sprain should have a careful exam of the 5th metatarsal to rule out a fracture. Local bruising, swelling, tenderness over the tuberosity to palpation, and pain with resisted eversion of the ankle are usually present in the case of a proximal metatarsal fracture. The proximal tuberosity acts as an attachment site for the peroneus brevis tendon and the lateral band of the plantar aponeurosis. These attachments can cause an avulsion force to the tuberosity. 1,2
Treatment of tuberosity fractures of the proximal 5th metatarsal are almost always treated nonoperatively. In the rare instance that the tuberosity fracture is severely displaced patients may require pin or screw fixation. Nondisplaced or minimally/moderately displaced tuberosity fractures can be treated in a walking boot weight bearing as tolerated. The fracture generally heals in 6-8 weeks. If patients have pain beyond 6-8 weeks they should be advised to avoid resisted eversion and plantar flexion of the ankle until the fracture becomes asymptomatic. A symptomatic nonunion, although rare, can be treated surgically with a single cancellous screw. 1,2References
1. Rosenberg GA, Sferra JJ. Treatment Strategies for Acute Fractures and Nonunions of the Proximal 5th Metatarsal. JAAOS 2000; 8: 332-338.
2. Metatarsal fractures. http://www.orthobullets.com. Accessed on 6/12/17.Incorrect
The majority of 5th metatarsal fractures occur proximally at the tuberosity. Proximal avulsion fractures are often called “pseudo Jones” fractures where a true jones fracture occurs at the metaphyseal-diaphyseal junction and area of poor circulation. Proximal fractures result from a forceful inversion of the foot in a plantar flexed position, or the same mechanism as a common lateral ankle sprain. For this reason, anyone who presents with a lateral ankle sprain should have a careful exam of the 5th metatarsal to rule out a fracture. Local bruising, swelling, tenderness over the tuberosity to palpation, and pain with resisted eversion of the ankle are usually present in the case of a proximal metatarsal fracture. The proximal tuberosity acts as an attachment site for the peroneus brevis tendon and the lateral band of the plantar aponeurosis. These attachments can cause an avulsion force to the tuberosity. 1,2
Treatment of tuberosity fractures of the proximal 5th metatarsal are almost always treated nonoperatively. In the rare instance that the tuberosity fracture is severely displaced patients may require pin or screw fixation. Nondisplaced or minimally/moderately displaced tuberosity fractures can be treated in a walking boot weight bearing as tolerated. The fracture generally heals in 6-8 weeks. If patients have pain beyond 6-8 weeks they should be advised to avoid resisted eversion and plantar flexion of the ankle until the fracture becomes asymptomatic. A symptomatic nonunion, although rare, can be treated surgically with a single cancellous screw. 1,2References
1. Rosenberg GA, Sferra JJ. Treatment Strategies for Acute Fractures and Nonunions of the Proximal 5th Metatarsal. JAAOS 2000; 8: 332-338.
2. Metatarsal fractures. http://www.orthobullets.com. Accessed on 6/12/17. -
Question 38 of 44
38. Question
An 18 year-old male presents to the office one week after sustaining a patella dislocation while playing football. He admits to falling awkwardly and noticing a deformity to his knee cap. He was able to reduce the patella with his hand and was able to walk after the reduction. He now presents with swelling and significant pain when flexing his right knee. Sunrise view x-ray (figure 1) shows a reduced patella located centrally within the trochlea groove and a small avulsion type fracture off the medial patella. What is the next best step in this patient’s treatment?Correct
The patient presents with swelling and pain after a patella dislocation. Given his swelling and x-ray findings, an MRI should be ordered to rule out an osteochondral defect and injury to the medial patellofemoral ligament (MPFL). A traumatic patella dislocation can break off an osteochondral fragment, generally off the lateral femoral condyle as the patella dislocates laterally. An osteochondral fragment can break off and float around the knee causing symptoms of a loose body. Symptoms include recurrent effusions and mechanical symptoms of catching or locking of the knee. The incidence of loose body formation after patella dislocation was found to be 22% in one study. 1
MPFL disruption may occur in up to 96% of patients that sustain a traumatic patella dislocation. The MPFL is the primary soft tissue stabilizer to lateral dislocation of the patella. The MPFL also helps the patella stay located centrally within the trochlea during knee flexion. MRI is the study of choice to determine if the MPFL is intact after a traumatic patella dislocation with a sensitivity of 85% and an accuracy of 80%. The standard of care for patella dislocations with MPFL disruption and without an OCD lesion is non-operative treatment. Conservative treatment is usually successfully with redislocation rates ranging from 15% to 44%. 2References
1. Farr J, Covell DJ, Lattermann C. Cartilage lesions in patellofemoral dislocations: Incidents/locations/when to treat. Sports Med Arthrosc. 2012 Sep; 20(3): 181–186.
2. Amin NH, Lynch TS, Patel RM, Patel N, Saluan P. Medial Patellafemoral ligament reconstruction. JBJS Reviews 2015; 3 (7): e3Incorrect
The patient presents with swelling and pain after a patella dislocation. Given his swelling and x-ray findings, an MRI should be ordered to rule out an osteochondral defect and injury to the medial patellofemoral ligament (MPFL). A traumatic patella dislocation can break off an osteochondral fragment, generally off the lateral femoral condyle as the patella dislocates laterally. An osteochondral fragment can break off and float around the knee causing symptoms of a loose body. Symptoms include recurrent effusions and mechanical symptoms of catching or locking of the knee. The incidence of loose body formation after patella dislocation was found to be 22% in one study. 1
MPFL disruption may occur in up to 96% of patients that sustain a traumatic patella dislocation. The MPFL is the primary soft tissue stabilizer to lateral dislocation of the patella. The MPFL also helps the patella stay located centrally within the trochlea during knee flexion. MRI is the study of choice to determine if the MPFL is intact after a traumatic patella dislocation with a sensitivity of 85% and an accuracy of 80%. The standard of care for patella dislocations with MPFL disruption and without an OCD lesion is non-operative treatment. Conservative treatment is usually successfully with redislocation rates ranging from 15% to 44%. 2References
1. Farr J, Covell DJ, Lattermann C. Cartilage lesions in patellofemoral dislocations: Incidents/locations/when to treat. Sports Med Arthrosc. 2012 Sep; 20(3): 181–186.
2. Amin NH, Lynch TS, Patel RM, Patel N, Saluan P. Medial Patellafemoral ligament reconstruction. JBJS Reviews 2015; 3 (7): e3 -
Question 39 of 44
39. Question
A 79 year-old male presents to your office with increasing thoracolumbar pain and difficulty straitening his back up. He denies radiating pain down the legs. He also denies a history of cancer or significant weight loss recently. A lateral x-ray and sagittal MRI show new wedge compression fractures at T11, T12, and L1. The heterogeneous signal in the marrow at these levels suggest subacute fractures. His DXA shows a T score of -3.7 at the lumbar spine (L1-L4), which has decreased by 25% since his last DXA 8 years ago. Labs show a calcium of 9.9 (nl 8.2-9.8 mg/dl), elevated alkaline phosphatase of 133 (nl 45-117 U/L), an intact parathyroid hormone (PTH) of 10.8 (normal 11.1-79.5 pg/ml), and a vitamin D of 28.7 (normal 30-100 ng/ml). He was also found to have an elevated calcium level in his 24-hour urine collection. What would be the next best step in the patient’s treatment?
Correct
A pathological fracture should always be considered in the differential diagnosis when a patient presents with a new compression fracture. This patient has severe osteoporosis at the lumbar spine which is likely the cause of his compression fractures. However, the cause of his poor bone quality is unclear. His slightly elevated serum calcium level, elevated urine calcium level, and elevated alkaline phosphatase is concerning for a destructive pathological lesion (when bone breaks down calcium and alkaline phosphatase is released). Multiple myeloma, which frequently presents with boney involvement, should be ruled out in this patient before assuming his osteoporosis is age related or caused by low vitamin D. Multiple myeloma is the most common primary bone malignancy. Up to 80% of newly diagnosed multiple myeloma patients will have boney involvement and the spine is the most common site involved. The malignant plasma cells of multiple myeloma invade bone and increase osteoclastic activity causing osteoporosis in the spine. MRI is the most sensitive diagnostic test to detect multiple myeloma, although lesions may not show on MRI early in the disease. Early detection of multiple myeloma may be accomplished through blood testing including a serum protein electrophoresis (SPEP). A monoclonal spike on SPEP is consistent with multiple myeloma. A bone marrow biopsy may be necessary to detect an increase in plasma cells in bone marrow which is diagnostic of multiple myeloma. Ordering a sestamibi for suspected parathyroid disease is incorrect because hyperparathyroidism would present with an elevated intact PTH. 1,2
References
1. Tosi, Patrizia. Diagnosis and Treatment of Bone Disease in Multiple Myeloma: Spotlight on Spinal Involvement. Scientifica. Volume 2013. Accessed 1/15/2013.
2. Multiple Myeloma. http://www.orthobullets.com. Accessed on 1/15/2018.Incorrect
A pathological fracture should always be considered in the differential diagnosis when a patient presents with a new compression fracture. This patient has severe osteoporosis at the lumbar spine which is likely the cause of his compression fractures. However, the cause of his poor bone quality is unclear. His slightly elevated serum calcium level, elevated urine calcium level, and elevated alkaline phosphatase is concerning for a destructive pathological lesion (when bone breaks down calcium and alkaline phosphatase is released). Multiple myeloma, which frequently presents with boney involvement, should be ruled out in this patient before assuming his osteoporosis is age related or caused by low vitamin D. Multiple myeloma is the most common primary bone malignancy. Up to 80% of newly diagnosed multiple myeloma patients will have boney involvement and the spine is the most common site involved. The malignant plasma cells of multiple myeloma invade bone and increase osteoclastic activity causing osteoporosis in the spine. MRI is the most sensitive diagnostic test to detect multiple myeloma, although lesions may not show on MRI early in the disease. Early detection of multiple myeloma may be accomplished through blood testing including a serum protein electrophoresis (SPEP). A monoclonal spike on SPEP is consistent with multiple myeloma. A bone marrow biopsy may be necessary to detect an increase in plasma cells in bone marrow which is diagnostic of multiple myeloma. Ordering a sestamibi for suspected parathyroid disease is incorrect because hyperparathyroidism would present with an elevated intact PTH. 1,2
References
1. Tosi, Patrizia. Diagnosis and Treatment of Bone Disease in Multiple Myeloma: Spotlight on Spinal Involvement. Scientifica. Volume 2013. Accessed 1/15/2013.
2. Multiple Myeloma. http://www.orthobullets.com. Accessed on 1/15/2018. -
Question 40 of 44
40. Question
A 29 year-old male presents to the emergency department with left leg and ankle pain after slipping on ice. He felt like the ankle twisted from under him and he has had difficulty bearing weight since. On exam the patient has pain over the proximal fibula and over the ankle at the deltoid ligament and anterior tibio-fibular ligament. He has diffuse swelling over both the medial and lateral ankle as well. AP and lateral x-rays of the tibia and fibula show an oblique fracture of the proximal fibula (figures 1 and 2). AP mortise and lateral x-rays of the ankle show no fracture or syndesmotic widening. What is the best treatment option for this patient?Correct
The patient’s proximal fibula fracture with ankle swelling is consistent with a syndesmotic disruption and Maisonneuve injury pattern. Maisonneuve fractures are often missed on initial evaluation which is why full length tibia-fibular x-rays should be performed after a significant ankle injury, particularly if ankle x-rays appear normal. It is important to understand the injury is not just an isolated fibular fracture. The injury generally occurs due to a torsional injury of the ankle when the force of the injury is transmitted along the interosseous ligament to the proximal fibula. The injury pattern also commonly causes disruption of the anterior tibio-fibular ligament and tear of the medial deltoid ligament. The mortise view x-ray of the ankle is the most important diagnostic modality to determine if a significant syndesmotic injury has occurred. Increased medial clear space, measured from the lateral boarder of the medial malleolus to the medial boarder of the talus on the mortise view, indicates a deltoid ligament injury. Medial clear space widening is also the most reliable indicator of a syndesmotic injury. Any evidence of syndesmosis widening on radiographs is an indication for surgery which includes syndesmosis screw fixation. If there is no syndesmosis widening, non-operative treatment is recommended. This includes a period of non-weight bearing for 4-6 weeks with progressive weight bearing to tolerance thereafter. Patients are often placed in a non-weight bearing boot or short leg cast during this time. Patients should be advised that the recovery time for syndesmotic injuries can be twice as long as a typical ankle sprain and can take up to 2-3 months before a return to normal function. 1,2
References
1. Pankovich AM. Maisonneuve fracture of the fibula. J Bone Joint Surg Am. 1976 Apr;58(3):337-42.
2. Syndesmosis Injury. http://www.orthobullets.com. Accessed on 4/29/17.Incorrect
The patient’s proximal fibula fracture with ankle swelling is consistent with a syndesmotic disruption and Maisonneuve injury pattern. Maisonneuve fractures are often missed on initial evaluation which is why full length tibia-fibular x-rays should be performed after a significant ankle injury, particularly if ankle x-rays appear normal. It is important to understand the injury is not just an isolated fibular fracture. The injury generally occurs due to a torsional injury of the ankle when the force of the injury is transmitted along the interosseous ligament to the proximal fibula. The injury pattern also commonly causes disruption of the anterior tibio-fibular ligament and tear of the medial deltoid ligament. The mortise view x-ray of the ankle is the most important diagnostic modality to determine if a significant syndesmotic injury has occurred. Increased medial clear space, measured from the lateral boarder of the medial malleolus to the medial boarder of the talus on the mortise view, indicates a deltoid ligament injury. Medial clear space widening is also the most reliable indicator of a syndesmotic injury. Any evidence of syndesmosis widening on radiographs is an indication for surgery which includes syndesmosis screw fixation. If there is no syndesmosis widening, non-operative treatment is recommended. This includes a period of non-weight bearing for 4-6 weeks with progressive weight bearing to tolerance thereafter. Patients are often placed in a non-weight bearing boot or short leg cast during this time. Patients should be advised that the recovery time for syndesmotic injuries can be twice as long as a typical ankle sprain and can take up to 2-3 months before a return to normal function. 1,2
References
1. Pankovich AM. Maisonneuve fracture of the fibula. J Bone Joint Surg Am. 1976 Apr;58(3):337-42.
2. Syndesmosis Injury. http://www.orthobullets.com. Accessed on 4/29/17. -
Question 41 of 44
41. Question
A 29 year-old male sustains multiple left sided rib fractures during a motor vehicle accident 5 days prior. You see the patient in surgical consultation for severe pain with deep inhalation and palpable motion of anterior segments of the left hemi thorax. Portable chest x-ray shows approximately 9 rib fractures on the left, of which ribs 4, 5, and 6 are noted to be a flail segment. He also has decreased pulmonary function tests. What is the best treatment option for this patient?Correct
Rib fractures are a commonly seen injury that presents in an emergency room setting. Most rib fractures are treat conservatively with observation but more severe injuries may be considered for surgical fixation. A flail chest, defined radiographically as 3 or more consecutive ribs with fractures in 2 or more places, is such an injury where surgical fixation is considered. Flail chest patients have a high risk of respiratory failure and mortality. Clinically a flail chest can be seen as paradoxal movement of the flail segment during respirations. In the past, treatment has commonly been pain management and positive pressure ventilation. However, rib fractures are now being increasingly fixed surgically with a 10 fold increase in surgical procedures over the last decade. The benefits of rib fixation over mechanical ventilation include a decrease in chest infections and chest wall deformity with an increase in pulmonary function tests. Open reduction and internal fixation also results in decreased ICU days, a trend toward a lower rate of pneumonia, and an increase in forced vital capacity. Indications for surgical fixation of a flail chest include persistent pain and decreased pulmonary function tests. Surgical fixation can be done non-urgently and the average time to fixation in many cases is around 6 days post injury. The most common technique for surgical fixation includes plates with corticol locking screws. 1,2
References
1. Tanaka H, Yukioka T, Yamaguti Y. Surgical stabilization of internal pneumatic stabilization? A prospective randomized study of management of severe flail chest patients. The Journal of Truma Injury, Infection, and Critical care. Volume 52 (4) 727-732.
2. de Moya M, Nirula R, Biffl W. Rib fixation: Who, What, and When? Trauma Surgery and Acute Care Open 2017; 2: 1-4.Incorrect
Rib fractures are a commonly seen injury that presents in an emergency room setting. Most rib fractures are treat conservatively with observation but more severe injuries may be considered for surgical fixation. A flail chest, defined radiographically as 3 or more consecutive ribs with fractures in 2 or more places, is such an injury where surgical fixation is considered. Flail chest patients have a high risk of respiratory failure and mortality. Clinically a flail chest can be seen as paradoxal movement of the flail segment during respirations. In the past, treatment has commonly been pain management and positive pressure ventilation. However, rib fractures are now being increasingly fixed surgically with a 10 fold increase in surgical procedures over the last decade. The benefits of rib fixation over mechanical ventilation include a decrease in chest infections and chest wall deformity with an increase in pulmonary function tests. Open reduction and internal fixation also results in decreased ICU days, a trend toward a lower rate of pneumonia, and an increase in forced vital capacity. Indications for surgical fixation of a flail chest include persistent pain and decreased pulmonary function tests. Surgical fixation can be done non-urgently and the average time to fixation in many cases is around 6 days post injury. The most common technique for surgical fixation includes plates with corticol locking screws. 1,2
References
1. Tanaka H, Yukioka T, Yamaguti Y. Surgical stabilization of internal pneumatic stabilization? A prospective randomized study of management of severe flail chest patients. The Journal of Truma Injury, Infection, and Critical care. Volume 52 (4) 727-732.
2. de Moya M, Nirula R, Biffl W. Rib fixation: Who, What, and When? Trauma Surgery and Acute Care Open 2017; 2: 1-4. -
Question 42 of 44
42. Question
A 19 year-old female complains of severe pain in her pelvis after a vaginal delivery of her first child. You see her a day after delivery and she is unable to get out of bed due to the pain. On exam she has no loss of sensation or motor weakness in the lower extremities. Pain to palpation is noted over the anterior pelvis. AP pelvis x-ray shows a 5 cm symphysis pubis diastasis without fracture. What is the next best step in treatment?Correct
The symphysis pubis is a fibrocartilaginous joint that connects the left and right superior pubic rami. The symphysis pubis widens during pregnancy as a normal response to child birth. The widening is generally 2-3 mm and rarely exceeds more than 1 cm. Progesterone and relaxin are natural hormones that help the strong supporting ligaments stretch as a normal physiological response to childbirth. Any widening usually regresses after delivery and rarely becomes symptomatic. Symphysis diastasis of over 1 cm represents complete disruption of the joint which is usually symptomatic. Complete disruption may be heard as a popping sound during delivery. A complete disruption causes pain over the symphysis pubis that is made worse with walking.
AP pelvis x-ray is the best study to diagnose symphysis pubis diastasis. A careful physical exam is also necessary to rule out sacroiliac joint (SI) involvement. The SI joint is usually not involved unless the diastasis exceeds 4 cm. Treatment is almost always non-operative. Patients may be weight bearing as tolerated and use a walker for support if needed. Pelvic binders and sleeping in a lateral position can also help with symptoms. Pain usually subsides in 8 weeks but can last as long as 8 months. The diastasis usually reduces over time and becomes asymptomatic. Surgery may be indicated in patients with persistent symptoms and a large diastasis.
References
1. Khorashadi L, Petscavage JM, Richardson ML. Postpartum symphysis pubis diastasis. Radiol Case Rep. 2011; 6(3): 542.
2. Neeta J, Sternberg L. Symphyseal Separation. Obstetrics & Gynecology: May 2005 – Volume 105 – Issue 5 – p 1229-1232.Incorrect
The symphysis pubis is a fibrocartilaginous joint that connects the left and right superior pubic rami. The symphysis pubis widens during pregnancy as a normal response to child birth. The widening is generally 2-3 mm and rarely exceeds more than 1 cm. Progesterone and relaxin are natural hormones that help the strong supporting ligaments stretch as a normal physiological response to childbirth. Any widening usually regresses after delivery and rarely becomes symptomatic. Symphysis diastasis of over 1 cm represents complete disruption of the joint which is usually symptomatic. Complete disruption may be heard as a popping sound during delivery. A complete disruption causes pain over the symphysis pubis that is made worse with walking.
AP pelvis x-ray is the best study to diagnose symphysis pubis diastasis. A careful physical exam is also necessary to rule out sacroiliac joint (SI) involvement. The SI joint is usually not involved unless the diastasis exceeds 4 cm. Treatment is almost always non-operative. Patients may be weight bearing as tolerated and use a walker for support if needed. Pelvic binders and sleeping in a lateral position can also help with symptoms. Pain usually subsides in 8 weeks but can last as long as 8 months. The diastasis usually reduces over time and becomes asymptomatic. Surgery may be indicated in patients with persistent symptoms and a large diastasis.
References
1. Khorashadi L, Petscavage JM, Richardson ML. Postpartum symphysis pubis diastasis. Radiol Case Rep. 2011; 6(3): 542.
2. Neeta J, Sternberg L. Symphyseal Separation. Obstetrics & Gynecology: May 2005 – Volume 105 – Issue 5 – p 1229-1232. -
Question 43 of 44
43. Question
A 66 year-old male presents to the office with “years” of on and off lower back pain and stiffness. His stiffness is worse in the morning and seems to be aggravated by cold weather. His pain is predominately located across his mid to lower back. He denies radiation of pain to the lower extremities. AP and lateral x-rays show non-marginal syndesmophytes at three successive levels of the lumbar spine. What is the most likely diagnosis?Correct
Diffuse idiopathic skeletal hyperostosis (DISH) is an idiopathic condition characterized by calcification of the tendons and ligaments around the spine. The tendons and ligaments can progressively calcify and turn into bone. DISH can also occur in several other locations in the body including the knees, elbows, foot and ankles, and shoulders. DISH is a relatively common condition occurring in up to 25% of persons over 50 years of age. Many patients with DISH are asymptomatic and the condition is often found incidentally on x-ray. Diagnostic criteria includes osteophytes along the anteriorlateral boarders of at least four contiguous segments. Other x-ray findings include preservation of disc height and absence of degenerative changes and ankylosis at the facet and sacroiliac joints. The thoracic spine is the most frequently involved followed by the lumbar spine and cervical spine. DISH patients are more prone to fracture and spinal instability as a result of stiff spinal ankylosis. DISH patients with even minor neck trauma should be worked up closely with x-ray and CT to rule out fracture. Treatment of DISH involves NSAIDS, physical therapy for core strengthening and flexibility, corset bracing, and bisphosphonate therapy. Surgery is not indicated for the treatment of DISH, and fortunately pain is usually maintained well with conservative treatment. 1,2
References
1. Belanger TA, Rowe DE. Diffuse Idiopathic Skeletal Hyperostosis: Musculoskeletal Manifestations. JAAOS 2001. Vol 9 (4). p258-267.
2. Diffuse idiopathic skeletal hyperostosis (DISH). http://www.orthobullets.com. Accessed 11/22/2017.Incorrect
Diffuse idiopathic skeletal hyperostosis (DISH) is an idiopathic condition characterized by calcification of the tendons and ligaments around the spine. The tendons and ligaments can progressively calcify and turn into bone. DISH can also occur in several other locations in the body including the knees, elbows, foot and ankles, and shoulders. DISH is a relatively common condition occurring in up to 25% of persons over 50 years of age. Many patients with DISH are asymptomatic and the condition is often found incidentally on x-ray. Diagnostic criteria includes osteophytes along the anteriorlateral boarders of at least four contiguous segments. Other x-ray findings include preservation of disc height and absence of degenerative changes and ankylosis at the facet and sacroiliac joints. The thoracic spine is the most frequently involved followed by the lumbar spine and cervical spine. DISH patients are more prone to fracture and spinal instability as a result of stiff spinal ankylosis. DISH patients with even minor neck trauma should be worked up closely with x-ray and CT to rule out fracture. Treatment of DISH involves NSAIDS, physical therapy for core strengthening and flexibility, corset bracing, and bisphosphonate therapy. Surgery is not indicated for the treatment of DISH, and fortunately pain is usually maintained well with conservative treatment. 1,2
References
1. Belanger TA, Rowe DE. Diffuse Idiopathic Skeletal Hyperostosis: Musculoskeletal Manifestations. JAAOS 2001. Vol 9 (4). p258-267.
2. Diffuse idiopathic skeletal hyperostosis (DISH). http://www.orthobullets.com. Accessed 11/22/2017. -
Question 44 of 44
44. Question
A 56 year-old male presents to the office with “years” of left foot pain. The pain has become progressively worse over the last month and is now bothering him every day. He notices the pain more when he his walking up and down stairs and the pain seems to be relieved with rest. AP and lateral view x-rays of the left foot show arthritis at the tarsometatarsal (TMT) joint. What is the next best treatment option?Correct
The midfoot, or tarsometatarsal (TMT) joint, bears the bodies weight as the joint helps propel the foot forward during ambulation. Inflammation of the midfoot can cause pain every time the foot pushes off the ground. Arthritis of the TMT joint can be a disabling condition causing chronic pain with ambulation. The most common cause of arthritis in the midfoot is a prior history of trauma. Post-traumatic arthritis can be caused by a fall or crush injury. A common mechanism of injury is a motor vehicle accident when the foot is plantar flexed during impact. A Lis franc injury, or a fracture dislocation of the midfoot, can also result in midfoot arthritis years after injury. As the supporting ligaments of the midfoot are injured, TMT joints become unstable, hypermobile, and articular cartilage subsequently wears over time. 1,2
Pain with TMT joint arthritis can be reproduced by having the patient perform a single leg stance while trying to push off with the affected foot. X-ray findings will include joint space narrowing at the 1rst, 2nd, and 3rd metatarsal bases and their articulations with the medial, middle, and lateral cuneiforms. The primary goal of treatment for TMT joint arthritis is to unload the joint with immobilization. A carbon fiber insole orthotic placed in a shoe is usually the first line treatment to immobilize the midfoot. NSAIDs, along with a period of ice and rest, helps reduce inflammation and pain. Steroid injections done under fluoroscopic guidance are helpful for both diagnostic and therapeutic purposes for persistent pain. TMT joint arthrodesis is the preferred surgical option if all conservative measures fail. 1,2
References
1. Rao S, Nawoczenski DA, Baumhauer JF. Midfoot Arthritis: Nonoperative options and Decision making for fusion. Techniques in Foot and Ankle Surgery 2008. 7 (3): 188-195
2. Midfoot Arthritis. http://www.orthobullets.com. Accessed on 11/22/17.Incorrect
The midfoot, or tarsometatarsal (TMT) joint, bears the bodies weight as the joint helps propel the foot forward during ambulation. Inflammation of the midfoot can cause pain every time the foot pushes off the ground. Arthritis of the TMT joint can be a disabling condition causing chronic pain with ambulation. The most common cause of arthritis in the midfoot is a prior history of trauma. Post-traumatic arthritis can be caused by a fall or crush injury. A common mechanism of injury is a motor vehicle accident when the foot is plantar flexed during impact. A Lis franc injury, or a fracture dislocation of the midfoot, can also result in midfoot arthritis years after injury. As the supporting ligaments of the midfoot are injured, TMT joints become unstable, hypermobile, and articular cartilage subsequently wears over time. 1,2
Pain with TMT joint arthritis can be reproduced by having the patient perform a single leg stance while trying to push off with the affected foot. X-ray findings will include joint space narrowing at the 1rst, 2nd, and 3rd metatarsal bases and their articulations with the medial, middle, and lateral cuneiforms. The primary goal of treatment for TMT joint arthritis is to unload the joint with immobilization. A carbon fiber insole orthotic placed in a shoe is usually the first line treatment to immobilize the midfoot. NSAIDs, along with a period of ice and rest, helps reduce inflammation and pain. Steroid injections done under fluoroscopic guidance are helpful for both diagnostic and therapeutic purposes for persistent pain. TMT joint arthrodesis is the preferred surgical option if all conservative measures fail. 1,2
References
1. Rao S, Nawoczenski DA, Baumhauer JF. Midfoot Arthritis: Nonoperative options and Decision making for fusion. Techniques in Foot and Ankle Surgery 2008. 7 (3): 188-195
2. Midfoot Arthritis. http://www.orthobullets.com. Accessed on 11/22/17.